SlideShare a Scribd company logo
1 of 99
Download to read offline
Tanvir Touhid
Puzzles
Always write down whatever you’re studying
1.
Draw diagrams for road, network etc. type puzzles
2.
If it wants sequences of correct answers, take 1 constraint and then
eliminate one option
3.
Use the rough page given behind the question paper in the exam hall
to do puzzles.
4.
Tanvir Touhid
Zebra Puzzle #1
Eight persons F, G, H, I, J, K, L and M attend 8 different seminars in four different months:
January, April, November and December of the same year. Each seminar was conducted on
either 15th or 25th of the month.
G attends the seminar on the 15th of a month which has only 30 days.
M and J both attend their seminars in the month before G, but not in the month of April.
M attends the seminar after J.
No one attends the seminar after F.
Only one person attends the seminar between M and K.
Both H and L attend the seminar on the 15th of different months.
L does not attend the seminar in December.
L and I attend the seminar in the same month.
Tanvir Touhid
11. Who among the following attends the seminar on 15th Nov?
(A). K (B). L (C). K (D). I (E). H
12. Which of the following combinations of Month-Date-Person is correct?
(A). January – 15 – G (B). January – 25 – K (C). Nov – 25 – I
(D). April – 15 – F (E). April – 25 – F
13. Which of the following statements is true with respect to the given arrangement?
(A) G attends the seminar in November
(B) H attends the seminar immediately after I.
(C) M attends the seminar immediately before K.
(D) K attends the seminar in November
(E) None of the given statements is true
14. Who among the following attends a seminar in December?
(A). F (B). M (C). G (D). K (E). J
15. How many people attend seminars between G and H?
(A). One (B). Two (C). Three (D). Four (E). More than Four Tanvir Touhid
Zebra Puzzle #2
(i) Five friends P, Q, R. S and T travelled to five different cities of Chennai, Calcutta, Delhi,
Bangalore and
Hyderabad by five different modes of transport of Bus. Train, Aeroplane, Car and Boat from
Mumbai.
(ii) The person who traveled to Delhi did not travel by boat.
(iii) R went to Bangalore by car and Q went to Calcutta by aeroplane.
(iv) S travelled by boat whereas T travelled by train.
(v) Mumbai is not connected by bus to Delhi and Chennai.
Tanvir Touhid
Zebra Puzzle #2
1. Which of the following combinations of person and mode is not correct?
(A) P — Bus (B) Q — Aeroplane (C) R — Car (D) S — Boat (E) T — Aeroplane
2. Which of the following combinations is true for S?
(A) Delhi — Bus
(B) Chennai — Bus
(C) Chennai — Boat
(D) Data inadequate
(E) None of these
3. Which of the following combinations of place and mode is not correct?
(A) Delhi — Bus
(B) Calcutta — Aeroplane
(C) Bangalore — Car
(D) Chennai — Boat
(E) Hyderabad — Bus Tanvir Touhid
Zebra Puzzle #2
4. The person travelling to Delhi went by which of the following modes?
(A) Bus (B) Train (C) Aeroplane (D) Car (E) Boat
5. Who among the following travelled to Delhi?
(A) R (B) S (C) T (D) Data inadequate (E) None of these
Tanvir Touhid
Two circular dials of exactly the same size are
pg 256
A chemist has exactly four unlabeled bottles
pg 216
A cryptanalyst must translate into letters all of the digits
pg 963
Seven pieces of equipment-an air compressor
pg 884
An archaeologist is excavating a system of paths used by traders
pg 489
In a city, the public transportation system consists
of one subway line and one bus line
pg 817
Internal audits in the Goodcrop Corporation
pg 818
During its manufacture any widget
pg 578
Difficult
Puzzles
Six horses-with the names "Greatness," "
pg 763
A map is being prepared that will represent
pg 687
A team of art historians has the task
pg 549
Seven offices in an office building are to be painted
pg 348
A student is planning his class schedule
pg 293
A foundation has appointed two review panels
pg 1043
Researchers are testing numerous water
pg 1042
A certain kind of traditional cloth is woven
pg 1022
Tanvir Touhid
Tanvir Touhid
Critical Reasoning
Tanvir Touhid
Inference; are un-stated partial conclusions that can be drawn from the
given premise. They do not contain the central idea, but they lead
to/support the central idea.
• An inference can serve as a link in the line of reasoning but is not the
same as conclusion. A conclusion
invariably addresses the central idea of the stimulus whereas inference
serves only to support the conclusion
• There can be many inferences; such as immediate inference, followed by
final inference, before it leads to
conclusion.
Tanvir Touhid
Premise #1: Students who get seven hours of sleep at night tend to be
more alert the next day than those who don’t get seven hours of sleep
Premise #2: The ability to get good scores in any competitive exam
depends on one’s level of alertness.
• Inference: From the above two we can infer that, “if you are a student
(may not be true for others) you are likely to be more alert next morning if
you get seven hours of sleep”
• Conclusion: From the above two we can conclude that, “student (may
not be true for others) wishing to do well in competitive exams should try
and get at least seven hours of sleep on day before the exam)
Tanvir Touhid
Approaching / Solving Critical Reasoning Questions
1 Read the stimulus, little swiftly
2 Read the question & re-read the stimulus, little steadily
3 Identify the conclusion**
4 Separate the evidence from the conclusion
5 Re-arrange the premise and conclusion to get clear line of
reasoning
6 Pre-phrase the answer
7 Attack the question
Tanvir Touhid
Jimmy is a doctor (premise)
All doctors go to medical school.(premise)
So jimmy went to medical school (conclusion)
Basic
Tanvir Touhid
A. Assumptions
• An assumption is merely an unstated (implied) premise.
• In logically correct arguments which contain an assumption, the premise +
assumption = conclusion.
• An assumption bridges the gap between argument’s stated premises and
conclusion.
• Remember, since the assumption is an UNSTATED premise, any answer choice that
comes from the passage to support your assumption is necessarily incorrect.
• For assumption questions, find the conclusion and determine which answer choice
needs to be true for a conclusion to be valid.(It must be a statement that completely
supports the conclusion)
Types
Tanvir Touhid
CORRECT ANSWER CHOICES:
• Will be supporter or defender
• Supporters help to link unrelated information presented in the stimulus and fill
logical gaps
• Defenders eliminate possibilities of weakness and attack to the
stimulus/conclusion.
That means, assumption questions:
Tanvir Touhid
Assumption Problem #1
For several years, Nighttime News attracted
fewer viewers than World News, which
broadcasts its show at the same time as
Nighttime News. Recently,the producers of
Nighttime News added personal interest
stories and increased coverage of sports
and weather. The two programs now have a
roughly equal number of viewers. Clearly,
the recent programming changes
persuaded viewers to switch from World
News to Nighttime News.
Which of the following is an assumption
on which the author relies?
(D) There are other possible causes for an increase in
the number of viewers of Nighttime News, including
a recent ad campaign that aired on many local
affiliates.
(A) Viewers are more interested in sports and
weather than in personal interest stories.
B) The programming content of Nighttime
News is more closely aligned with the interests
of the overall audience than is the content of
World News.
(C) Some World News viewers liked the new
Nighttime News programming better than they
liked the World News programming.
(E) The quality of World News will remain
constant even if Nighttime News improves.
Tanvir Touhid
Assumption Problem #1
For several years, Nighttime News attracted
fewer viewers than World News, which
broadcasts its show at the same time as
Nighttime News. Recently,the producers of
Nighttime News added personal interest
stories and increased coverage of sports
and weather. The two programs no.w have a
roughly equal number of viewers. Clearly,
the recent programming changes
persuaded viewers to switch from World
News to Nighttime News.
Which of the following is an assumption
on which the author relies?
(D) There are other possible causes for an increase in
the number of viewers of Nighttime News, including
a recent ad campaign that aired on many local
affiliates.
(A) Viewers are more interested in sports and
weather than in personal interest stories.
B) The programming content of Nighttime
News is more closely aligned with the interests
of the overall audience than is the content of
World News.
(C) Some World News viewers liked the new
Nighttime News programming better than
they liked the World News programming.
(E) The quality of World News will remain
constant even if Nighttime News improves.
Tanvir Touhid
Assumption Problem #2
Although the ratio of physicians to total
population is about the same in the United
States and Canada, the United States has 33
percent more surgeons per capita. Clearly,
this is the reason people in the United
States undergo 40 percent more operations
per capita than do Canadians.
The explanation given above rests on an
assumption that:
(D) General practitioners in the United States do not
as a rule examine a who is a candidate for surgery
before sending the patient to a surgeon.
(A) patients in the United States do not have a
greater need for surgery than do patients in
Canada.
(B) the population of the United States is not
larger than that of Canada.
(C) United States patients sometimes travel to
Canada for certain kinds of surgery.
(E) There are no unnecessary surgical
operations performed in Canada.
Tanvir Touhid
Assumption Problem #2
Although the ratio of physicians to total
population is about the same in the United
States and Canada, the United States has 33
percent more surgeons per capita. Clearly,
this is the reason people in the United
States undergo 40 percent more operations
per capita than do Canadians.
The explanation given above rests on an
assumption that:
(D) General practitioners in the United States do not
as a rule examine a who is a candidate for surgery
before sending the patient to a surgeon.
(A) patients in the United States do not have a
greater need for surgery than do patients in
Canada.
(B) the population of the United States is not
larger than that of Canada.
(C) United States patients sometimes travel to
Canada for certain kinds of surgery.
(E) There are no unnecessary surgical
operations performed in Canada.
Tanvir Touhid
Assumption Problem #3
Ramirez: The film industry claims that
pirated DVDs, which are usually cheaper
than legitimate DVDs and become available
well before a film's official DVD release
date, adversely affect its bottom line. But
the industry should note what the spread of
piracy indicates: consumers want lower
prices and faster DVD releases. Lowering
prices of DVDs and releasing them sooner
would mitigate piracy's negative effect on
film industry profits.
The argument above relies on which of
the following assumptions?
D. Some current sellers of pirated DVDs would likely
discontinue their businesses if legitimate DVDs
were released faster and priced lower.
A. Releasing legitimate DVDs earlier would not cause
any reduction in the revenue the film industry
receives from the films' theatrical release.
B. Some people who would otherwise purchase
pirated DVDs would be willing to purchase
legitimate DVDs if they were less expensive and
released earlier than they are now.
C. The film industry will in the future be able to
produce DVDs more cheaply than is currently the
case.
E. Current purchasers of pirated DVDs are aware
that those DVDs are not authorized by the film
industry.
Tanvir Touhid
Assumption Problem #3
Ramirez: The film industry claims that
pirated DVDs, which are usually cheaper
than legitimate DVDs and become available
well before a film's official DVD release
date, adversely affect its bottom line. But
the industry should note what the spread of
piracy indicates: consumers want lower
prices and faster DVD releases. Lowering
prices of DVDs and releasing them sooner
would mitigate piracy's negative effect on
film industry profits.
The argument above relies on which of
the following assumptions?
D. Some current sellers of pirated DVDs would likely
discontinue their businesses if legitimate DVDs
were released faster and priced lower.
A. Releasing legitimate DVDs earlier would not cause
any reduction in the revenue the film industry
receives from the films' theatrical release.
B. Some people who would otherwise purchase
pirated DVDs would be willing to purchase
legitimate DVDs if they were less expensive and
released earlier than they are now.
C. The film industry will in the future be able to
produce DVDs more cheaply than is currently the
case.
E. Current purchasers of pirated DVDs are aware
that those DVDs are not authorized by the film
industry.
Tanvir Touhid
Assumption Problem #4
Because no employee wants to be
associated with bad news in the eyes of a
superior, information about serious
problems at lower levels is progressively
softened and distorted as it goes up each
step in the management hierarchy. The
chief executive is therefore, less well
informed about problems at lower levels
than are his or her subordinates at those
levels.
The conclusion drawn above is based on the
assumption that
The conclusion drawn above is based on the
assumption that
(A) problems should be solved at levels in the
management hierarchy at which they occur
(B) employees should be rewarded for accurately
reporting problems to their superiors
(C) problems-solving ability is more important at higher
levels than it is at lower levels of the management
hierarchy
(D) chief executives obtain information about problems
at lower levels from the no source other than their
subordinates
(E) some employees are more concerned about truth
than about the way they are perceived by their
superiors
Tanvir Touhid
Assumption Problem #4
Because no employee wants to be
associated with bad news in the eyes of a
superior, information about serious
problems at lower levels is progressively
softened and distorted as it goes up each
step in the management hierarchy. The
chief executive is therefore, less well
informed about problems at lower levels
than are his or her subordinates at those
levels.
The conclusion drawn above is based on the
assumption that
The conclusion drawn above is based on the
assumption that
(A) problems should be solved at levels in the
management hierarchy at which they occur
(B) employees should be rewarded for accurately
reporting problems to their superiors
(C) problems-solving ability is more important at higher
levels than it is at lower levels of the management
hierarchy
(D) chief executives obtain information about
problems at lower levels from the no source other
than their subordinates
(E) some employees are more concerned about truth
than about the way they are perceived by their
superiors
Tanvir Touhid
Assumption Problem #5
Colleges in Tycho City have failed to prepare
their students for the business world. A
recent study revealed that the majority of
college graduates in Tycho City could not
write a simple business letter.
Which of the following, if true, would provide
additional evidence in support of the claim
above?
(A) A majority of students attending colleges in Tycho
City are business majors.
B) The State College in neighboring Twyla Township has
recently improved its business program by adding
courses in business writing.
C) Most Tycho City college graduates move outside the
Tycho City after they graduate.
D) Most Tycho City college students live in on-campus
dormitories.
E) The majority of college graduates living in Tycho City
received their college degrees from institutions located
in Tycho city.
Tanvir Touhid
Assumption Problem #5
Colleges in Tycho City have failed to prepare
their students for the business world. A
recent study revealed that the majority of
college graduates in Tycho City could not
write a simple business letter.
Which of the following, if true, would provide
additional evidence in support of the claim
above?
(A) A majority of students attending colleges in Tycho
City are business majors.
B) The State College in neighboring Twyla Township has
recently improved its business program by adding
courses in business writing.
C) Most Tycho City college graduates move outside the
Tycho City after they graduate.
D) Most Tycho City college students live in on-campus
dormitories.
E) The majority of college graduates living in Tycho
City received their college degrees from institutions
located in Tycho city.
Tanvir Touhid
B. Strengthen the Argument
Identify the conclusion—this is what you are trying to strengthen!
Find the logical gap and fix it with additional information. This is the ONLY type of
GMAT question where additional information (outside of the question) can/should be
used.
Correct answers to this question type will:
• Connect evidence with conclusion better
• Make conclusion stronger.
• Strengthen the evidence with new information (perhaps an assumption is needed to
make the argument work)
Types
Tanvir Touhid
Strengthen Problem #1
In an effort to make college education more
affordable, the government of Grendania
plans to subsidize college education by
paying up to fifteen percent of college
tuition for any Grendanian who attends
college full time. However, rather than make
college more affordable for students,
implementing this plan will serve only to
increase the revenue of universities.
Which of the following most clearly
supports the conclusion above?
A. The government plans to leave in place
programs that fund research engaged in by
university professors.
B. Universities in Grendania base the levels of their
tuition on the amounts that people are willing to
pay.
C. When college education is more affordable,
more people choose to attend college full time.
D. In Grendania, the cost of attending college has
been increasing at a rate much faster than the
general rate of price inflation.
E. Even if the average college tuition in Grendania
were fifteen percent lower, many people would
still have difficulty affording to attend college.
Tanvir Touhid
Strengthen Problem #1
In an effort to make college education more
affordable, the government of Grendania
plans to subsidize college education by
paying up to fifteen percent of college
tuition for any Grendanian who attends
college full time. However, rather than make
college more affordable for students,
implementing this plan will serve only to
increase the revenue of universities.
Which of the following most clearly
supports the conclusion above?
A. The government plans to leave in place
programs that fund research engaged in by
university professors.
B. Universities in Grendania base the levels of
their tuition on the amounts that people are
willing to pay.
C. When college education is more affordable,
more people choose to attend college full time.
D. In Grendania, the cost of attending college has
been increasing at a rate much faster than the
general rate of price inflation.
E. Even if the average college tuition in Grendania
were fifteen percent lower, many people would
still have difficulty affording to attend college.
Tanvir Touhid
Strengthen Problem #2
Mr. Blatt: Expert consultants are sought
after by management because they help
executives make better decisions. That is
why they are worth the substantial fees
they charge.
Ms. Fring: Nonsense. Expert consultants are
hired in order to enable executives to avoid
responsibility. The more the experts cost,
the more they can be blamed when things
go wrong.
Which one of the following, if it
occurred, would be the strongest
evidence favoring Ms. Fring’s opinion?
(A) A company that is trying to decide whether to
move its manufacturing plant hires an expensive
expert to conduct a cost/benefit analysis.
(B) Two competing companies faced with very
similar problems adopt different solutions, one
with the help of a consultant, one without.
(C) A successful firm of expert consultants seeks to
increase its volume of business by reducing its
fees, but its volume of business drops.
(D) An expert consultant builds up a successful
business by charging clients a substantial
percentage of the amount an independent
assessor judges that the consultant saved the
company.
(E) A company follows a consultant’s advice to
open two new stores, but both stores are only
marginally profitable at first.
Tanvir Touhid
Strengthen Problem #2
Mr. Blatt: Expert consultants are sought
after by management because they help
executives make better decisions. That is
why they are worth the substantial fees
they charge.
Ms. Fring: Nonsense. Expert consultants are
hired in order to enable executives to avoid
responsibility. The more the experts cost,
the more they can be blamed when things
go wrong.
Which one of the following, if it
occurred, would be the strongest
evidence favoring Ms. Fring’s opinion?
(A) A company that is trying to decide whether to
move its manufacturing plant hires an expensive
expert to conduct a cost/benefit analysis.
(B) Two competing companies faced with very
similar problems adopt different solutions, one
with the help of a consultant, one without.
(C) A successful firm of expert consultants seeks
to increase its volume of business by reducing its
fees, but its volume of business drops.
(D) An expert consultant builds up a successful
business by charging clients a substantial
percentage of the amount an independent
assessor judges that the consultant saved the
company.
(E) A company follows a consultant’s advice to
open two new stores, but both stores are only
marginally profitable at first.
Tanvir Touhid
Strengthen Problem #3
One summer, floods covered low-lying
garlic fields situated in a region with a large
mosquito population. Since mosquitoes lay
their eggs in standing water, flooded fields
would normally attract mosquitoes, yet no
mosquitoes were found in the fields. Diallyl
sulfide, a major component of garlic, is
known to repel several species of insects,
including mosquitoes, so it is likely that
diallyl sulfide from the garlic repelled the
mosquitoes.
Which of the following, if true, most
strengthens the argument?
(A) Diallyl sulfide is also found in onions but at
concentrations lower than in garlic.
(B) The mosquito population of the region as a
whole was significantly smaller during the year in
which the flooding took place than it had been in
previous years.
(C) By the end of the summer, most of the garlic
plants in the flooded fields had been killed by
waterborne fungi.
(D) Many insect species not repelled by diallyl
sulfide were found in the flooded garlic fields
throughout the summer.
(E) Mosquitoes are known to be susceptible to
toxins in plants other than garlic, such as
marigolds.
Tanvir Touhid
Strengthen Problem #3
One summer, floods covered low-lying
garlic fields situated in a region with a large
mosquito population. Since mosquitoes lay
their eggs in standing water, flooded fields
would normally attract mosquitoes, yet no
mosquitoes were found in the fields. Diallyl
sulfide, a major component of garlic, is
known to repel several species of insects,
including mosquitoes, so it is likely that
diallyl sulfide from the garlic repelled the
mosquitoes.
Which of the following, if true, most
strengthens the argument?
(A) Diallyl sulfide is also found in onions but at
concentrations lower than in garlic.
(B) The mosquito population of the region as a
whole was significantly smaller during the year in
which the flooding took place than it had been in
previous years.
(C) By the end of the summer, most of the garlic
plants in the flooded fields had been killed by
waterborne fungi.
(D) Many insect species not repelled by diallyl
sulfide were found in the flooded garlic fields
throughout the summer.
(E) Mosquitoes are known to be susceptible to
toxins in plants other than garlic, such as
marigolds.
Tanvir Touhid
C. Weaken The Argument
To solve these questions, you first need to identify the premise and the conclusion. In this
question type, we assume an answer choice presented to be true – even if it introduces new
information (obviously, the information has to be relevant to the stimulus)
ANSWER CHOICE QUALIFICATION:
• Should rebuke the conclusion of the stimulus
• Answer choices are taken to be true, even if there is new information provided.
• Will either break down causality or show an obvious error in reasoning in formation of the
conclusion
CORRECT ANSWER CHOICES:
• Will point out an obvious reason for the illogical conclusion
• Enumerate a wrong generalization
• Point out improper comparisons between two scenarios that the author assumed
Types
Tanvir Touhid
Weaken Problem #1
Teenagers are often priced out of the labor
market by the government-mandated
minimum-wage level because employers
cannot afford to pay that much for extra
help. Therefore, if Congress institutes a
subminimum wage, a new lower legal wage
for teenagers, the teenage unemployment
rate, which has been rising since 1960, will
no longer increase.
Which of the following, if true, would
most weaken the argument above?
(A) Since 1960 the teenage unemployment rate
has risen when the minimum wage has risen.
(B) Since 1960 the teenage unemployment rate
has risen even when the minimum wage remained
constant.
(C) Employers often hire extra help during holiday
and warm weather seasons.
(D) The teenage unemployment rate rose more
quickly in the 1970's than it did in the 1960's.
(E) The teenage unemployment rate has
occasionally declined in the years since 1960.
Tanvir Touhid
Weaken Problem #1
Teenagers are often priced out of the labor
market by the government-mandated
minimum-wage level because employers
cannot afford to pay that much for extra
help. Therefore, if Congress institutes a
subminimum wage, a new lower legal wage
for teenagers, the teenage unemployment
rate, which has been rising since 1960, will
no longer increase.
Which of the following, if true, would
most weaken the argument above?
(A) Since 1960 the teenage unemployment rate
has risen when the minimum wage has risen.
(B) Since 1960 the teenage unemployment rate
has risen even when the minimum wage remained
constant.
(C) Employers often hire extra help during holiday
and warm weather seasons.
(D) The teenage unemployment rate rose more
quickly in the 1970's than it did in the 1960's.
(E) The teenage unemployment rate has
occasionally declined in the years since 1960.
Tanvir Touhid
Weaken Problem #2
Opponents of laws that require automobile
drivers and passengers to wear seat belts
argue that in a free society people have the
right to take risks as long as people do not
harm others as a result of taking the risks.
As a result, they conclude that it should be
each person’s decision whether or not to
wear a seat belt.
Which of the following, if true, most
seriously weakens the conclusion drawn
above?
(A) Many new cars are built with seat belts that
automatically fasten when someone sits in the
front seat.
(B) Automobile insurance rates for all automobile
owners are higher because of the need to pay for
the increased injuries or deaths of people not
wearing seat belts.
(C) Passengers in airplanes are required to wear
seat belts during takeoffs and landings.
(D) The rate of automobile fatalities in states that
do not have mandatory seat belt laws is greater
than the rate of fatalities in states that do have
such laws.
(E) In automobile accidents, a greater number of
passengers who do not wear seat belts are injured
than are passengers who do wear seat belts.
Tanvir Touhid
Weaken Problem #2
Opponents of laws that require automobile
drivers and passengers to wear seat belts
argue that in a free society people have the
right to take risks as long as people do not
harm others as a result of taking the risks.
As a result, they conclude that it should be
each person’s decision whether or not to
wear a seat belt.
Which of the following, if true, most
seriously weakens the conclusion drawn
above?
(A) Many new cars are built with seat belts that
automatically fasten when someone sits in the
front seat.
(B) Automobile insurance rates for all automobile
owners are higher because of the need to pay for
the increased injuries or deaths of people not
wearing seat belts.
(C) Passengers in airplanes are required to wear
seat belts during takeoffs and landings.
(D) The rate of automobile fatalities in states that
do not have mandatory seat belt laws is greater
than the rate of fatalities in states that do have
such laws.
(E) In automobile accidents, a greater number of
passengers who do not wear seat belts are injured
than are passengers who do wear seat belts.
Tanvir Touhid
Weaken Problem #3
A study of marital relationships in which
one partner's sleeping and waking cycles
differ from those of the other partner
reveals that such couples share fewer
activities with each other and have more
violent arguments than do couples in a
relationship in which both partners follow
the same sleeping and waking patterns.
Thus, mismatched sleeping and waking
cycles can seriously jeopardize a marriage.
Which of the following, if true, most
seriously weakens the argument above?
A) Married couples in which both spouses follow
the same sleeping and waking patterns also
occasionally have arguments that can jeopardize
the couple's marriage.
(B) The sleeping and waking cycles of individuals
tend to vary from season to season.
(C) The individuals who have sleeping and waking
cycles that differ significantly from those of their
spouses tend to argue little with colleagues at
work.
(D) People in unhappy marriages have been found
to express hostility by adopting a different
sleeping and waking cycle from that of their
spouses.
(E) According to a recent study, most people's
sleeping and waking cycles can be controlled and
modified easily.
Tanvir Touhid
Weaken Problem #3
A study of marital relationships in which
one partner's sleeping and waking cycles
differ from those of the other partner
reveals that such couples share fewer
activities with each other and have more
violent arguments than do couples in a
relationship in which both partners follow
the same sleeping and waking patterns.
Thus, mismatched sleeping and waking
cycles can seriously jeopardize a marriage.
Which of the following, if true, most
seriously weakens the argument above?
A) Married couples in which both spouses follow
the same sleeping and waking patterns also
occasionally have arguments that can jeopardize
the couple's marriage.
(B) The sleeping and waking cycles of individuals
tend to vary from season to season.
(C) The individuals who have sleeping and waking
cycles that differ significantly from those of their
spouses tend to argue little with colleagues at
work.
(D) People in unhappy marriages have been found
to express hostility by adopting a different
sleeping and waking cycle from that of their
spouses.
(E) According to a recent study, most people's
sleeping and waking cycles can be controlled and
modified easily.
Tanvir Touhid
D. MUST BE TRUE/ INFERENCE/ MAIN POINT/CONCLUSION
Consider the evidence, draw a conclusion.
An inference is an extension of an argument, not a necessary part of it.
A valid inference is a conclusion, but not necessarily the conclusion, of a set of statements.
For inference questions, determine which answer choice must absolutely, positively be true
based on what you’ve read.
• Pick the obvious answer choice.
• Avoid extreme answers (too strong or too weak)
CORRECT ANSWER CHOICES:
• Restatement of the conclusion
• Combination of one or more premises
• Should be the main point of the stimulus, not just a premise (for Main Point questions-
Repeat premises are wrong )
Types
Tanvir Touhid
Must-be-true Problem
Transportation expenses accounted for a
large portion of the total dollar amount
spent on trips for pleasure by residents of
the United States in 1997, and about half of
the total dollar amount spent on the
transportation was airfare. However, the
large majority of United States residents
who took trips for pleasure in 1997 did not
travel by airplane but used other means of
transportation.
If the statements above are true, which of
the following must also be true about
United States residents who took trips for
pleasure in 1997?
(A) Most of those who traveled by airplane did so
because the airfare to their destination was lower
than the cost of other available means of
transportation.
(B) Most of those who traveled by airplane did so
because other means of transportation to their
destinations were unavailable.
(C) Per mile traveled, those who traveled by
airplane tended to spend more on transportation
to their destination than did those who used other
means of transportation.
(D) Overall, people who did not travel by airplane
had lower average transportation expenses than
people who did.
(E) Those who traveled by airplane spent about as
much, on average, on other means of
transportation as they did on airfare.
Tanvir Touhid
Must-be-true Problem
Transportation expenses accounted for a
large portion of the total dollar amount
spent on trips for pleasure by residents of
the United States in 1997, and about half of
the total dollar amount spent on the
transportation was airfare. However, the
large majority of United States residents
who took trips for pleasure in 1997 did not
travel by airplane but used other means of
transportation.
If the statements above are true, which of
the following must also be true about
United States residents who took trips for
pleasure in 1997?
(A) Most of those who traveled by airplane did so
because the airfare to their destination was lower
than the cost of other available means of
transportation.
(B) Most of those who traveled by airplane did so
because other means of transportation to their
destinations were unavailable.
(C) Per mile traveled, those who traveled by
airplane tended to spend more on transportation
to their destination than did those who used other
means of transportation.
(D) Overall, people who did not travel by airplane
had lower average transportation expenses than
people who did.
(E) Those who traveled by airplane spent about as
much, on average, on other means of
transportation as they did on airfare.
Tanvir Touhid
Inference Problem
In many school districts, part-time teachers
are being laid off to save money. However, a
decrease in the number of teachers
generally results in a decrease in student
performance. Thus, the same school
districts that are laying off part-time
teachers to save money will eventually have
to hire more tutors to counteract
diminished student performance.
The author is arguing that
A. diminished student performance will keep
school districts from laying off part-time
teachers
B. laying off part-time teachers to save money
will result in other costs to the school districts
C. many part-time teachers who are laid off will
have to retrain for other types of jobs
D. school districts that are laying off teachers
will eventually rehire many of them
E. school districts will not save money by laying
off part-time teachers
Tanvir Touhid
Inference Problem
In many school districts, part-time teachers
are being laid off to save money. However, a
decrease in the number of teachers
generally results in a decrease in student
performance. Thus, the same school
districts that are laying off part-time
teachers to save money will eventually have
to hire more tutors to counteract
diminished student performance.
The author is arguing that
A. diminished student performance will keep
school districts from laying off part-time
teachers
B. laying off part-time teachers to save money
will result in other costs to the school districts
C. many part-time teachers who are laid off will
have to retrain for other types of jobs
D. school districts that are laying off teachers
will eventually rehire many of them
E. school districts will not save money by laying
off part-time teachers
Tanvir Touhid
Logical Flaw
Cotrell is, at best, able to write magazine
articles of average quality. The most
compelling piece of evidence for this are
those few of the numerous articles
submitted by Cotrell that are superior,
since Cotrell, who is incapable of writing an
article that is better than average, must
obviously have plagiarized superior ones.
The argument is most vulnerable to
criticism on which one of the following
grounds?
(A) It simply ignores the existence of potential
counterevidence.
(B) It generalizes from atypical occurrences.
(C) It presupposes what it seeks to establish
(D) It relies on the judgement of experts in a
matter to which their expertise is irrelevant
(E) It infers limits on ability from a few isolated
lapses in performance
Tanvir Touhid
Logical Flaw
Cotrell is, at best, able to write magazine
articles of average quality. The most
compelling piece of evidence for this are
those few of the numerous articles
submitted by Cotrell that are superior,
since Cotrell, who is incapable of writing an
article that is better than average, must
obviously have plagiarized superior ones.
The argument is most vulnerable to
criticism on which one of the following
grounds?
(A) It simply ignores the existence of potential
counterevidence.
(B) It generalizes from atypical occurrences.
(C) It presupposes what it seeks to establish
(D) It relies on the judgement of experts in a
matter to which their expertise is irrelevant
(E) It infers limits on ability from a few isolated
lapses in performance
Tanvir Touhid
Complete The Passage
Exporters in Country X are facing lower
revenues due to a shortage of the large
metal shipping containers in which they
send their goods by sea to other countries.
Fewer containers arrive in Country X due to
reductions in imports. This has meant lost
orders, costly delays, and a scramble for
alternatives, such as air freight, all of which
are costlier. Moreover, the revenues of
exporters in Country X will probably
continue to decline in the near future. This
is because other countries are likely to find
it increasingly unprofitable to export their
goods to Country X, and because __________.
A. production of shipping containers in
Country X is growing rapidly as a response to
the shortage
B. shipping companies are willing to move
containers from country to country only when
the containers are full
C. the cost of shipping alternatives such as air
freight is likely to stabilize in the near future
D. consumers in Country X are purchasing
more products than ever before
E. the worldwide demand for goods made in
Country X has only recently begun to rise after
a long decline
Tanvir Touhid
Complete The Passage
Exporters in Country X are facing lower
revenues due to a shortage of the large
metal shipping containers in which they
send their goods by sea to other countries.
Fewer containers arrive in Country X due to
reductions in imports. This has meant lost
orders, costly delays, and a scramble for
alternatives, such as air freight, all of which
are costlier. Moreover, the revenues of
exporters in Country X will probably
continue to decline in the near future. This
is because other countries are likely to find
it increasingly unprofitable to export their
goods to Country X, and because __________.
A. production of shipping containers in
Country X is growing rapidly as a response to
the shortage
B. shipping companies are willing to move
containers from country to country only when
the containers are full
C. the cost of shipping alternatives such as air
freight is likely to stabilize in the near future
D. consumers in Country X are purchasing
more products than ever before
E. the worldwide demand for goods made in
Country X has only recently begun to rise after
a long decline
Tanvir Touhid
Bold face CR
Public health expert: Increasing the urgency
of a public health message may be
counterproductive. In addition to irritating
the majority who already behave
responsibly, it may undermine all
government pronouncements on health by
convincing people that such messages are
overly cautious. And there is no reason to
believe that those who ignore measured
voices will listen to shouting.
The two sections in boldface play which of
the following roles in the public health
expert's argument?
(A) The first is a conclusion for which support is
provided. but is not the argument's main
conclusion; the second is an unsupported
premise supporting the arguments main
conclusion.
(B) The first is a premise supporting the only
explicit conclusion; so is the second.
(C) The first is the argument's main conclusion;
the second supports that conclusion and is
itself a conclusion for which support is
provided.
(D) The first is a premise supporting the
argument's only conclusion; the second is that
conclusion.
(E) The first is the argument‘s only explicit
conclusion; the second is a premise supporting
that conclusion.
Tanvir Touhid
Bold face CR
Public health expert: Increasing the
urgency of a public health message may be
counterproductive. In addition to irritating
the majority who already behave
responsibly, it may undermine all
government pronouncements on health by
convincing people that such messages are
overly cautious. And there is no reason to
believe that those who ignore measured
voices will listen to shouting.
The two sections in boldface play which of
the following roles in the public health
expert's argument?
(A) The first is a conclusion for which support is
provided. but is not the argument's main
conclusion; the second is an unsupported
premise supporting the arguments main
conclusion.
(B) The first is a premise supporting the only
explicit conclusion; so is the second.
(C) The first is the argument's main conclusion;
the second supports that conclusion and is
itself a conclusion for which support is
provided.
(D) The first is a premise supporting the
argument's only conclusion; the second is that
conclusion.
(E) The first is the argument‘s only explicit
conclusion; the second is a premise
supporting that conclusion.
Tanvir Touhid
E. Resolve the Paradox
To solve this type of question, look for a logically contradictory discrepancy.
• Often the correct answer will take a similar format (in terms of answer length or argument
structure).
Types
Tanvir Touhid
Resolve the Paradox #1
A certain type of insect trap uses a scented
lure to attract rose beetles into a plastic
bag from which it is difficult for them to
escape. If several of these traps are
installed in a backyard garden, the number
of rose beetles in the garden will be greatly
reduced. If only one trap is installed,
however, the number of rose beetles in the
garden will actually increase.
Which one of the following, if true, most
helps to resolve the apparent discrepancy?
(A) The scent of a single trap’s lure usually cannot be
detected throughout a backyard garden by rose beetles.
(B) Several traps are better able to catch a large number
of rose beetles than is one trap alone, since any rose
beetles that evade one trap are likely to encounter
another trap if there are several traps in the garden.
(C) When there are several traps in a garden, they each
capture fewer rose beetles than any single trap would if
it were the only trap in the garden.
(D) The presence of any traps in a backyard garden will
attract more rose beetles than one trap can catch, but
several traps will not attract significantly more rose
beetles to a garden than one trap will.
(E) When there is only one trap in the garden, the plastic
bag quickly becomes filled to capacity, allowing some
rose beetles to escape.
Tanvir Touhid
Resolve the Paradox #1
A certain type of insect trap uses a scented
lure to attract rose beetles into a plastic
bag from which it is difficult for them to
escape. If several of these traps are
installed in a backyard garden, the number
of rose beetles in the garden will be greatly
reduced. If only one trap is installed,
however, the number of rose beetles in the
garden will actually increase.
Which one of the following, if true, most
helps to resolve the apparent discrepancy?
(A) The scent of a single trap’s lure usually cannot be
detected throughout a backyard garden by rose beetles.
(B) Several traps are better able to catch a large number
of rose beetles than is one trap alone, since any rose
beetles that evade one trap are likely to encounter
another trap if there are several traps in the garden.
(C) When there are several traps in a garden, they each
capture fewer rose beetles than any single trap would if
it were the only trap in the garden.
(D) The presence of any traps in a backyard garden will
attract more rose beetles than one trap can catch, but
several traps will not attract significantly more rose
beetles to a garden than one trap will.
(E) When there is only one trap in the garden, the plastic
bag quickly becomes filled to capacity, allowing some
rose beetles to escape.
Tanvir Touhid
Resolve the Paradox #2
Technological improvements and reduced
equipment costs have made converting
solar energy directly into electricity far
more cost-efficient in the last decade.
However, the threshold of economic
viability for solar power (that is, the price
per barrel to which oil would have to rise in
order for new solar power plants to be more
economical than new oil-fired power plants)
is unchanged at thirty-five dollars.
Which of the following, if true, does most to
help explain why the increased cost-
efficiency of solar power has not decreased
its threshold of economic viability?
(A) The cost of oil has fallen dramatically.
(B) The reduction in the cost of solar-power equipment
has occurred despite increased raw material costs for
that equipment.
(C) Technological changes have increased the efficiency
of oil-fired power plants.
(D) Most electricity is generated by coal-fired or
nuclear, rather than oil-fired, power plants.
(E) When the price of oil increases, reserves of oil not
previously worth exploiting become economically
viable.
Tanvir Touhid
Resolve the Paradox #2
Technological improvements and reduced
equipment costs have made converting
solar energy directly into electricity far
more cost-efficient in the last decade.
However, the threshold of economic
viability for solar power (that is, the price
per barrel to which oil would have to rise in
order for new solar power plants to be more
economical than new oil-fired power plants)
is unchanged at thirty-five dollars.
Which of the following, if true, does most to
help explain why the increased cost-
efficiency of solar power has not decreased
its threshold of economic viability?
(A) The cost of oil has fallen dramatically.
(B) The reduction in the cost of solar-power equipment
has occurred despite increased raw material costs for
that equipment.
(C) Technological changes have increased the
efficiency of oil-fired power plants.
(D) Most electricity is generated by coal-fired or
nuclear, rather than oil-fired, power plants.
(E) When the price of oil increases, reserves of oil not
previously worth exploiting become economically
viable.
Tanvir Touhid
F. Evaluate the Argument
-The question stem will likely use some form of the word “evaluate,” “determine,” or “useful
/ important to know.”
-Find the conclusion and identify the main supporting premise. If you can articulate any
gaps between the main premise and the conclusion, do so. Think of the conclusion as the
author’s hypothesis; your job is to find the answer choice that can test that hypothesis. The
answers will provide a “whether or not” type of statement, indicating information that might
or might not be so. The correct answer will be able to both slightly strengthen and slightly
weaken the hypothesis, depending upon whether the information in that choice is true or is
not true.
Types
Tanvir Touhid
Evaluate the Argument #1
Psychologist: In a survey, several hundred
volunteers rated their own levels of self-
control and their overall life satisfaction.
The volunteers who rated themselves as
having better self-control also reported
greater satisfaction with their lives. This
suggests that self-control is one factor that
helps people avoid situations likely to
produce dissatisfaction.
In order to assess the strength of the
psychologist's argument, it would be most
helpful to know whether
A. people typically rate themselves as having
significantly better self-control than expert
psychological assessments would rate them as having
B. people's perceptions of how satisfied they are with
their lives could be affected by factors of which they
are unaware
C. there is a high level of self-control that tends to
reduce overall life satisfaction
D. people's ratings of their overall satisfaction with their
lives tend to temporarily decrease in situations likely to
produce dissatisfaction
E. feelings of dissatisfaction significantly interfere with
people's ability to exercise self-control
Tanvir Touhid
Evaluate the Argument #1
Psychologist: In a survey, several hundred
volunteers rated their own levels of self-
control and their overall life satisfaction.
The volunteers who rated themselves as
having better self-control also reported
greater satisfaction with their lives. This
suggests that self-control is one factor that
helps people avoid situations likely to
produce dissatisfaction.
In order to assess the strength of the
psychologist's argument, it would be most
helpful to know whether
A. people typically rate themselves as having
significantly better self-control than expert
psychological assessments would rate them as having
B. people's perceptions of how satisfied they are with
their lives could be affected by factors of which they
are unaware
C. there is a high level of self-control that tends to
reduce overall life satisfaction
D. people's ratings of their overall satisfaction with their
lives tend to temporarily decrease in situations likely to
produce dissatisfaction
E. feelings of dissatisfaction significantly interfere with
people's ability to exercise self-control
Tanvir Touhid
Evaluate the Argument #2
Most of the world's supply of uranium
currently comes from the mines. It is
possible to extract uranium from seawater,
but the cost of doing so is greater than the
price that Uranium fetches on the world
market. Therefore, until the cost of
extracting uranium from seawater can
somehow be reduced, this method of
obtaining uranium is unlikely to be
commercially viable.
Which of the following would it be most
useful to determine in evaluating the
argument?
(A) Whether the uranium in deposits on land is rapidly
being depleted
(B) Whether most uranium is used near where it is mined
(C) Whether there are any technological advances that
show promise of reducing the costs of extracting
uranium from seawater
(D) Whether the total amount of Uranium in seawater is
significantly greater than the total amount of uranium
on land
(E) Whether uranium can be extracted from freshwater
at a cost similar to the cost of extracting it from
seawater
Tanvir Touhid
Evaluate the Argument #2
Most of the world's supply of uranium
currently comes from the mines. It is
possible to extract uranium from seawater,
but the cost of doing so is greater than the
price that Uranium fetches on the world
market. Therefore, until the cost of
extracting uranium from seawater can
somehow be reduced, this method of
obtaining uranium is unlikely to be
commercially viable.
Which of the following would it be most
useful to determine in evaluating the
argument?
(A) Whether the uranium in deposits on land is rapidly
being depleted
(B) Whether most uranium is used near where it is mined
(C) Whether there are any technological advances that
show promise of reducing the costs of extracting
uranium from seawater
(D) Whether the total amount of Uranium in seawater is
significantly greater than the total amount of uranium
on land
(E) Whether uranium can be extracted from freshwater
at a cost similar to the cost of extracting it from
seawater
Tanvir Touhid
Data Sufficiency
XY=?
(1) X-Y=2
(2) X+Y=6
A – (1) ALONE is sufficient, but (2) alone is not sufficient.
B – (2) ALONE is sufficient, but (1) alone is not sufficient.
C – TOGETHER are sufficient, but NEITHER ALONE is sufficient.
D – EACH ALONE is sufficient.
E – NEITHER ALONE NOR TOGETHER is the statements sufficient.
Tanvir Touhid
#1- Is |x - y| > |x + y|?
(1) x^2 - y^2 = 9
(2) x - y = 2
Tanvir Touhid
#1- Is |x - y| > |x + y|?
(1) x^2 - y^2 = 9
(2) x - y = 2
A – (1) ALONE is sufficient, but (2) alone is not sufficient.
B – (2) ALONE is sufficient, but (1) alone is not sufficient.
C – TOGETHER are sufficient, but NEITHER ALONE is sufficient.
D – EACH ALONE is sufficient.
E – NEITHER ALONE NOR TOGETHER is the statements sufficient.
Tanvir Touhid
#2- If x is a positive number less than 10, is z greater than
the average (arithmetic mean) of x and 10?
(1) On the number line, z is closer to 10 than it is to x.
(2) z = 5x
Tanvir Touhid
#2- If x is a positive number less than 10, is z greater than
the average (arithmetic mean) of x and 10?
(1) On the number line, z is closer to 10 than it is to x.
(2) z = 5x
A – (1) ALONE is sufficient, but (2) alone is not sufficient.
B – (2) ALONE is sufficient, but (1) alone is not sufficient.
C – TOGETHER are sufficient, but NEITHER ALONE is sufficient.
D – EACH ALONE is sufficient.
E – NEITHER ALONE NOR TOGETHER is the statements sufficient.
Tanvir Touhid
#3- Amir sold a pen and a pencil in his shop. If both pen and pencil are sold at a
certain profit, then the profit earned on pen is what percent of the profit earned on
pencil?
(1) Cost price of pen and cost price of pencil is in the ratio of 5:1.
(2) The Pen is sold for $3.5 while the pencil is sold for $0.7.
Tanvir Touhid
#3- Amir sold a pen and a pencil in his shop. If both pen and pencil are sold at a
certain profit, then the profit earned on pen is what percent of the profit earned on
pencil?
(1) Cost price of pen and cost price of pencil is in the ratio of 5:1.
(2) The Pen is sold for $3.5 while the pencil is sold for $0.7.
A – (1) ALONE is sufficient, but (2) alone is not sufficient.
B – (2) ALONE is sufficient, but (1) alone is not sufficient.
C – TOGETHER are sufficient, but NEITHER ALONE is sufficient.
D – EACH ALONE is sufficient.
E – NEITHER ALONE NOR TOGETHER is the statements sufficient.
Tanvir Touhid
#4- A team is composed of exactly 4 executives and 3 associates. How many
such teams can be formed?
(1) If two fewer executives were available for selection there would be an equal
number of executives and associates available for selection.
(2) If two more associates were available for selection exactly 10 different teams of
3 associates could be selected.
Tanvir Touhid
#4- A team is composed of exactly 4 executives and 3 associates. How many
such teams can be formed?
(1) If two fewer executives were available for selection there would be an equal
number of executives and associates available for selection.
(2) If two more associates were available for selection exactly 10 different teams of
3 associates could be selected.
A – (1) ALONE is sufficient, but (2) alone is not sufficient.
B – (2) ALONE is sufficient, but (1) alone is not sufficient.
C – TOGETHER are sufficient, but NEITHER ALONE is sufficient.
D – EACH ALONE is sufficient.
E – NEITHER ALONE NOR TOGETHER is the statements sufficient.
Tanvir Touhid
#5- During a 40-mile trip, Marla traveled at an average speed of x miles per hour
for the first y miles of the trip and and at an average speed of 1.25x miles per
hour for the last 40 - y miles of the trip. The time that Marla took to travel the 40
miles was what percent of the time it would have taken her if she had traveled at
an average speed of x miles per hour for the entire trip?
(1) x = 48.
(2) y = 20.
Tanvir Touhid
#5- During a 40-mile trip, Marla traveled at an average speed of x miles per hour
for the first y miles of the trip and and at an average speed of 1.25x miles per
hour for the last 40 - y miles of the trip. The time that Marla took to travel the 40
miles was what percent of the time it would have taken her if she had traveled at
an average speed of x miles per hour for the entire trip?
(1) x = 48.
(2) y = 20.
A – (1) ALONE is sufficient, but (2) alone is not sufficient.
B – (2) ALONE is sufficient, but (1) alone is not sufficient.
C – TOGETHER are sufficient, but NEITHER ALONE is sufficient.
D – EACH ALONE is sufficient.
E – NEITHER ALONE NOR TOGETHER is the statements sufficient.
Tanvir Touhid
#6- If 75 percent of the guests at a certain banquet ordered dessert, what
percent of the guests ordered coffee?
(1) 60 percent of the guests who ordered dessert also ordered coffee.
(2) 90 percent of the guests who ordered coffee also ordered dessert.
Tanvir Touhid
#6- If 75 percent of the guests at a certain banquet ordered dessert, what
percent of the guests ordered coffee?
(1) 60 percent of the guests who ordered dessert also ordered coffee.
(2) 90 percent of the guests who ordered coffee also ordered dessert.
A – (1) ALONE is sufficient, but (2) alone is not sufficient.
B – (2) ALONE is sufficient, but (1) alone is not sufficient.
C – TOGETHER are sufficient, but NEITHER ALONE is sufficient.
D – EACH ALONE is sufficient.
E – NEITHER ALONE NOR TOGETHER is the statements sufficient.
Tanvir Touhid
#7- The average (arithmetic mean) of a list of positive numbers is what percent
of the sum of the numbers?
(1) The sum of the numbers in the list is 150.
(2) There are 25 numbers in the list
Tanvir Touhid
#7- The average (arithmetic mean) of a list of positive numbers is what percent
of the sum of the numbers?
(1) The sum of the numbers in the list is 150.
(2) There are 25 numbers in the list
A – (1) ALONE is sufficient, but (2) alone is not sufficient.
B – (2) ALONE is sufficient, but (1) alone is not sufficient.
C – TOGETHER are sufficient, but NEITHER ALONE is sufficient.
D – EACH ALONE is sufficient.
E – NEITHER ALONE NOR TOGETHER is the statements sufficient.
Tanvir Touhid
#8- Is x > 10^10 ?
(1) x > 2^34
(2) x = 2^35
Tanvir Touhid
#8- Is x > 10^10 ?
(1) x > 2^34
(2) x = 2^35
A – (1) ALONE is sufficient, but (2) alone is not sufficient.
B – (2) ALONE is sufficient, but (1) alone is not sufficient.
C – TOGETHER are sufficient, but NEITHER ALONE is sufficient.
D – EACH ALONE is sufficient.
E – NEITHER ALONE NOR TOGETHER is the statements sufficient.
Tanvir Touhid
#9- If x is an integer, is 2^x a factor of 12! ?
(1) x is the sum of two distinct single-digit prime numbers.
(2) 0 < x < 11
Tanvir Touhid
#9- If x is an integer, is 2^x a factor of 12! ?
(1) x is the sum of two distinct single-digit prime numbers.
(2) 0 < x < 11
A – (1) ALONE is sufficient, but (2) alone is not sufficient.
B – (2) ALONE is sufficient, but (1) alone is not sufficient.
C – TOGETHER are sufficient, but NEITHER ALONE is sufficient.
D – EACH ALONE is sufficient.
E – NEITHER ALONE NOR TOGETHER is the statements sufficient.
Tanvir Touhid
#10-If x and y are integers and xy ≠ 0, is x - y > 0 ?
(1) x/y < 1/2
(2)
Tanvir Touhid
#10-If x and y are integers and xy ≠ 0, is x - y > 0 ?
(1) x/y < 1/2
(2)
A – (1) ALONE is sufficient, but (2) alone is not sufficient.
B – (2) ALONE is sufficient, but (1) alone is not sufficient.
C – TOGETHER are sufficient, but NEITHER ALONE is sufficient.
D – EACH ALONE is sufficient.
E – NEITHER ALONE NOR TOGETHER is the statements sufficient.
Tanvir Touhid
#11-On the number line, the distance between x and y is
greater than the distance between x and z. Does z lie between
x and y on the number line?
(1) xyz < 0
(2) xy < 0
Tanvir Touhid
#11-On the number line, the distance between x and y is
greater than the distance between x and z. Does z lie between
x and y on the number line?
(1) xyz < 0
(2) xy < 0
A – (1) ALONE is sufficient, but (2) alone is not sufficient.
B – (2) ALONE is sufficient, but (1) alone is not sufficient.
C – TOGETHER are sufficient, but NEITHER ALONE is sufficient.
D – EACH ALONE is sufficient.
E – NEITHER ALONE NOR TOGETHER is the statements
sufficient.
Tanvir Touhid
#12-A paint mixture was formed by mixing exactly 3 colors of paint. By
volume, the mixture was x% blue paint, y% green paint, and z% red paint.
If exactly 1 gallon of blue paint and 3 gallons of red paint were used, how
many gallons of green paint were used?
(1) x = y
(2) z = 60
Tanvir Touhid
#12-A paint mixture was formed by mixing exactly 3 colors of paint. By
volume, the mixture was x% blue paint, y% green paint, and z% red paint.
If exactly 1 gallon of blue paint and 3 gallons of red paint were used, how
many gallons of green paint were used?
(1) x = y
(2) z = 60
A – (1) ALONE is sufficient, but (2) alone is not sufficient.
B – (2) ALONE is sufficient, but (1) alone is not sufficient.
C – TOGETHER are sufficient, but NEITHER ALONE is sufficient.
D – EACH ALONE is sufficient.
E – NEITHER ALONE NOR TOGETHER is the statements sufficient.
Tanvir Touhid
#13-The positive integers x, y, and z are such that x is a factor of y and y is
a factor of z. Is z even?
(1) xz is even
(2) y is even.
Tanvir Touhid
#13-The positive integers x, y, and z are such that x is a factor of y and y is
a factor of z. Is z even?
(1) xz is even
(2) y is even.
A – (1) ALONE is sufficient, but (2) alone is not sufficient.
B – (2) ALONE is sufficient, but (1) alone is not sufficient.
C – TOGETHER are sufficient, but NEITHER ALONE is sufficient.
D – EACH ALONE is sufficient.
E – NEITHER ALONE NOR TOGETHER is the statements sufficient.
Tanvir Touhid
#14-Water in a flooded basement is to be pumped into an empty right
circular cylindrical container. If the height of the container is 250 cm, is
the volume of the container sufficient for it to hold all the water?
(1) The area of the flooded portion of the basement is 20 square meters.
(2) The circumference of the container is 100 centimeters
Tanvir Touhid
#14-Water in a flooded basement is to be pumped into an empty right
circular cylindrical container. If the height of the container is 250 cm, is
the volume of the container sufficient for it to hold all the water?
(1) The area of the flooded portion of the basement is 20 square meters.
(2) The circumference of the container is 100 centimeters
A – (1) ALONE is sufficient, but (2) alone is not sufficient.
B – (2) ALONE is sufficient, but (1) alone is not sufficient.
C – TOGETHER are sufficient, but NEITHER ALONE is sufficient.
D – EACH ALONE is sufficient.
E – NEITHER ALONE NOR TOGETHER is the statements
sufficient.
Tanvir Touhid
#15-A real estate agent sold 6 houses in Junction City and 3 houses in Uniontown. For each
house he sold, he earned a commission of 7% of the selling price of the house. Was the total
of his commissions for the houses he sold in Junction City greater than the total of his
commissions for the houses he sold in Uniontown?
(1) The average (arithmetic mean) selling price of the 3 houses in Uniontown was more than
twice the average selling price of the 6 houses in Junction City.
(2) The selling price of each of the 3 houses in Uniontown was more than twice the selling
price of the most expensive of the 6 houses in Junction City.
Tanvir Touhid
#15-A real estate agent sold 6 houses in Junction City and 3 houses in Uniontown. For each
house he sold, he earned a commission of 7% of the selling price of the house. Was the total
of his commissions for the houses he sold in Junction City greater than the total of his
commissions for the houses he sold in Uniontown?
(1) The average (arithmetic mean) selling price of the 3 houses in Uniontown was more than
twice the average selling price of the 6 houses in Junction City.
(2) The selling price of each of the 3 houses in Uniontown was more than twice the selling
price of the most expensive of the 6 houses in Junction City.
A – (1) ALONE is sufficient, but (2) alone is not sufficient.
B – (2) ALONE is sufficient, but (1) alone is not sufficient.
C – TOGETHER are sufficient, but NEITHER ALONE is sufficient.
D – EACH ALONE is sufficient.
E – NEITHER ALONE NOR TOGETHER is the statements sufficient.
Tanvir Touhid
#16-In a survey of retailers, what percent had purchased computers for
business purposes?
(1) 85 percent of the retailers surveyed who owned their own store had
purchased computers for business purposes.
(2) 40 percent of the retailers surveyed owned their own store.
Tanvir Touhid
#16-In a survey of retailers, what percent had purchased computers for
business purposes?
(1) 85 percent of the retailers surveyed who owned their own store had
purchased computers for business purposes.
(2) 40 percent of the retailers surveyed owned their own store.
A – (1) ALONE is sufficient, but (2) alone is not sufficient.
B – (2) ALONE is sufficient, but (1) alone is not sufficient.
C – TOGETHER are sufficient, but NEITHER ALONE is sufficient.
D – EACH ALONE is sufficient.
E – NEITHER ALONE NOR TOGETHER is the statements
sufficient.
Tanvir Touhid
#17-Seven different numbers are selected from the integers 1 to 100, and
each number is divided by 7. What is the sum of the remainders?
(1) The range of the seven remainders is 6.
(2) The seven numbers selected are consecutive integers.
Tanvir Touhid
#17-Seven different numbers are selected from the integers 1 to 100, and
each number is divided by 7. What is the sum of the remainders?
(1) The range of the seven remainders is 6.
(2) The seven numbers selected are consecutive integers.
A – (1) ALONE is sufficient, but (2) alone is not sufficient.
B – (2) ALONE is sufficient, but (1) alone is not sufficient.
C – TOGETHER are sufficient, but NEITHER ALONE is sufficient.
D – EACH ALONE is sufficient.
E – NEITHER ALONE NOR TOGETHER is the statements sufficient.
Tanvir Touhid
#18-Is x > 1?
(1) (x+1)(|x|−1)>0
(2) |x|<5
Tanvir Touhid
#18-Is x > 1?
(1) (x+1)(|x|−1)>0
(2) |x|<5
A – (1) ALONE is sufficient, but (2) alone is not sufficient.
B – (2) ALONE is sufficient, but (1) alone is not sufficient.
C – TOGETHER are sufficient, but NEITHER ALONE is sufficient.
D – EACH ALONE is sufficient.
E – NEITHER ALONE NOR TOGETHER is the statements sufficient.
Tanvir Touhid
Thank You

More Related Content

Similar to Ultimate_Analytical_by_Tanvir_Touhid.pdf

Latihan un-smk-2012-b-ingg-nonteknik
Latihan un-smk-2012-b-ingg-nonteknikLatihan un-smk-2012-b-ingg-nonteknik
Latihan un-smk-2012-b-ingg-nonteknik
Nafis Kurtubi
 
Question 1 (1 point)Which of the following isa characteristi.docx
Question 1 (1 point)Which of the following isa characteristi.docxQuestion 1 (1 point)Which of the following isa characteristi.docx
Question 1 (1 point)Which of the following isa characteristi.docx
amrit47
 
Đề thi thử Ôn thi Tiếng Anh vào Cao Đẳng , Đại học năm 2013 - Đề 105
Đề thi thử  Ôn thi Tiếng Anh vào Cao Đẳng , Đại học năm 2013 - Đề 105Đề thi thử  Ôn thi Tiếng Anh vào Cao Đẳng , Đại học năm 2013 - Đề 105
Đề thi thử Ôn thi Tiếng Anh vào Cao Đẳng , Đại học năm 2013 - Đề 105
phamnhakb
 

Similar to Ultimate_Analytical_by_Tanvir_Touhid.pdf (20)

Aptitude reasoning subject_specialisatio
Aptitude reasoning subject_specialisatioAptitude reasoning subject_specialisatio
Aptitude reasoning subject_specialisatio
 
Đề thi học sinh giỏi quốc gia môn Anh 2015
Đề thi học sinh giỏi quốc gia môn Anh 2015Đề thi học sinh giỏi quốc gia môn Anh 2015
Đề thi học sinh giỏi quốc gia môn Anh 2015
 
Communication_Studies_Paper_1s_2014-2020_with_solutions_5.pdf
Communication_Studies_Paper_1s_2014-2020_with_solutions_5.pdfCommunication_Studies_Paper_1s_2014-2020_with_solutions_5.pdf
Communication_Studies_Paper_1s_2014-2020_with_solutions_5.pdf
 
Latihan un-smk-2012-b-ingg-nonteknik
Latihan un-smk-2012-b-ingg-nonteknikLatihan un-smk-2012-b-ingg-nonteknik
Latihan un-smk-2012-b-ingg-nonteknik
 
2014 exam
2014 exam2014 exam
2014 exam
 
Question 1 (1 point)Which of the following isa characteristi.docx
Question 1 (1 point)Which of the following isa characteristi.docxQuestion 1 (1 point)Which of the following isa characteristi.docx
Question 1 (1 point)Which of the following isa characteristi.docx
 
294 bai tap co loi giai chi tiet
294 bai tap co loi giai chi tiet294 bai tap co loi giai chi tiet
294 bai tap co loi giai chi tiet
 
DILR Math.pdf
DILR Math.pdfDILR Math.pdf
DILR Math.pdf
 
Paragraph Organization Test.pdf
Paragraph Organization Test.pdfParagraph Organization Test.pdf
Paragraph Organization Test.pdf
 
ETS 2022 Test 6.pdf
ETS 2022 Test 6.pdfETS 2022 Test 6.pdf
ETS 2022 Test 6.pdf
 
Đề thi thử Ôn thi Tiếng Anh vào Cao Đẳng , Đại học năm 2013 - Đề 105
Đề thi thử  Ôn thi Tiếng Anh vào Cao Đẳng , Đại học năm 2013 - Đề 105Đề thi thử  Ôn thi Tiếng Anh vào Cao Đẳng , Đại học năm 2013 - Đề 105
Đề thi thử Ôn thi Tiếng Anh vào Cao Đẳng , Đại học năm 2013 - Đề 105
 
Tenth class-state syllabus-model paper-em-ap-social studies
Tenth class-state syllabus-model paper-em-ap-social studiesTenth class-state syllabus-model paper-em-ap-social studies
Tenth class-state syllabus-model paper-em-ap-social studies
 
Wipro paper
Wipro paperWipro paper
Wipro paper
 
Gre overview
Gre overview Gre overview
Gre overview
 
TNPSC Group II Examination Answer Key - General English
TNPSC Group II Examination Answer Key - General EnglishTNPSC Group II Examination Answer Key - General English
TNPSC Group II Examination Answer Key - General English
 
2019 Question paper by NCHMCT for NHTET Entrance exam
2019 Question paper by NCHMCT for NHTET Entrance exam2019 Question paper by NCHMCT for NHTET Entrance exam
2019 Question paper by NCHMCT for NHTET Entrance exam
 
GMAT Intro Workshop
GMAT Intro WorkshopGMAT Intro Workshop
GMAT Intro Workshop
 
UPSC_CSAT_previous_year_question_paper_Prelims_GS-II_2018.pdf
UPSC_CSAT_previous_year_question_paper_Prelims_GS-II_2018.pdfUPSC_CSAT_previous_year_question_paper_Prelims_GS-II_2018.pdf
UPSC_CSAT_previous_year_question_paper_Prelims_GS-II_2018.pdf
 
DILR.pdf
DILR.pdfDILR.pdf
DILR.pdf
 
Cts aptitude questio papers
Cts aptitude questio papersCts aptitude questio papers
Cts aptitude questio papers
 

Recently uploaded

The Abortion pills for sale in Qatar@Doha [+27737758557] []Deira Dubai Kuwait
The Abortion pills for sale in Qatar@Doha [+27737758557] []Deira Dubai KuwaitThe Abortion pills for sale in Qatar@Doha [+27737758557] []Deira Dubai Kuwait
The Abortion pills for sale in Qatar@Doha [+27737758557] []Deira Dubai Kuwait
daisycvs
 

Recently uploaded (20)

Berhampur Call Girl Just Call 8084732287 Top Class Call Girl Service Available
Berhampur Call Girl Just Call 8084732287 Top Class Call Girl Service AvailableBerhampur Call Girl Just Call 8084732287 Top Class Call Girl Service Available
Berhampur Call Girl Just Call 8084732287 Top Class Call Girl Service Available
 
Lundin Gold - Q1 2024 Conference Call Presentation (Revised)
Lundin Gold - Q1 2024 Conference Call Presentation (Revised)Lundin Gold - Q1 2024 Conference Call Presentation (Revised)
Lundin Gold - Q1 2024 Conference Call Presentation (Revised)
 
HomeRoots Pitch Deck | Investor Insights | April 2024
HomeRoots Pitch Deck | Investor Insights | April 2024HomeRoots Pitch Deck | Investor Insights | April 2024
HomeRoots Pitch Deck | Investor Insights | April 2024
 
Getting Real with AI - Columbus DAW - May 2024 - Nick Woo from AlignAI
Getting Real with AI - Columbus DAW - May 2024 - Nick Woo from AlignAIGetting Real with AI - Columbus DAW - May 2024 - Nick Woo from AlignAI
Getting Real with AI - Columbus DAW - May 2024 - Nick Woo from AlignAI
 
The Abortion pills for sale in Qatar@Doha [+27737758557] []Deira Dubai Kuwait
The Abortion pills for sale in Qatar@Doha [+27737758557] []Deira Dubai KuwaitThe Abortion pills for sale in Qatar@Doha [+27737758557] []Deira Dubai Kuwait
The Abortion pills for sale in Qatar@Doha [+27737758557] []Deira Dubai Kuwait
 
ALWAR 💋 Call Girl 9827461493 Call Girls in Escort service book now
ALWAR 💋 Call Girl 9827461493 Call Girls in  Escort service book nowALWAR 💋 Call Girl 9827461493 Call Girls in  Escort service book now
ALWAR 💋 Call Girl 9827461493 Call Girls in Escort service book now
 
Nashik Call Girl Just Call 7091819311 Top Class Call Girl Service Available
Nashik Call Girl Just Call 7091819311 Top Class Call Girl Service AvailableNashik Call Girl Just Call 7091819311 Top Class Call Girl Service Available
Nashik Call Girl Just Call 7091819311 Top Class Call Girl Service Available
 
Falcon Invoice Discounting: The best investment platform in india for investors
Falcon Invoice Discounting: The best investment platform in india for investorsFalcon Invoice Discounting: The best investment platform in india for investors
Falcon Invoice Discounting: The best investment platform in india for investors
 
Phases of Negotiation .pptx
 Phases of Negotiation .pptx Phases of Negotiation .pptx
Phases of Negotiation .pptx
 
Escorts in Nungambakkam Phone 8250092165 Enjoy 24/7 Escort Service Enjoy Your...
Escorts in Nungambakkam Phone 8250092165 Enjoy 24/7 Escort Service Enjoy Your...Escorts in Nungambakkam Phone 8250092165 Enjoy 24/7 Escort Service Enjoy Your...
Escorts in Nungambakkam Phone 8250092165 Enjoy 24/7 Escort Service Enjoy Your...
 
Arti Languages Pre Seed Teaser Deck 2024.pdf
Arti Languages Pre Seed Teaser Deck 2024.pdfArti Languages Pre Seed Teaser Deck 2024.pdf
Arti Languages Pre Seed Teaser Deck 2024.pdf
 
PHX May 2024 Corporate Presentation Final
PHX May 2024 Corporate Presentation FinalPHX May 2024 Corporate Presentation Final
PHX May 2024 Corporate Presentation Final
 
Chennai Call Gril 80022//12248 Only For Sex And High Profile Best Gril Sex Av...
Chennai Call Gril 80022//12248 Only For Sex And High Profile Best Gril Sex Av...Chennai Call Gril 80022//12248 Only For Sex And High Profile Best Gril Sex Av...
Chennai Call Gril 80022//12248 Only For Sex And High Profile Best Gril Sex Av...
 
GUWAHATI 💋 Call Girl 9827461493 Call Girls in Escort service book now
GUWAHATI 💋 Call Girl 9827461493 Call Girls in  Escort service book nowGUWAHATI 💋 Call Girl 9827461493 Call Girls in  Escort service book now
GUWAHATI 💋 Call Girl 9827461493 Call Girls in Escort service book now
 
Nanded Call Girl Just Call 8084732287 Top Class Call Girl Service Available
Nanded Call Girl Just Call 8084732287 Top Class Call Girl Service AvailableNanded Call Girl Just Call 8084732287 Top Class Call Girl Service Available
Nanded Call Girl Just Call 8084732287 Top Class Call Girl Service Available
 
SEO Case Study: How I Increased SEO Traffic & Ranking by 50-60% in 6 Months
SEO Case Study: How I Increased SEO Traffic & Ranking by 50-60%  in 6 MonthsSEO Case Study: How I Increased SEO Traffic & Ranking by 50-60%  in 6 Months
SEO Case Study: How I Increased SEO Traffic & Ranking by 50-60% in 6 Months
 
Puri CALL GIRL ❤️8084732287❤️ CALL GIRLS IN ESCORT SERVICE WE ARW PROVIDING
Puri CALL GIRL ❤️8084732287❤️ CALL GIRLS IN ESCORT SERVICE WE ARW PROVIDINGPuri CALL GIRL ❤️8084732287❤️ CALL GIRLS IN ESCORT SERVICE WE ARW PROVIDING
Puri CALL GIRL ❤️8084732287❤️ CALL GIRLS IN ESCORT SERVICE WE ARW PROVIDING
 
Home Furnishings Ecommerce Platform Short Pitch 2024
Home Furnishings Ecommerce Platform Short Pitch 2024Home Furnishings Ecommerce Platform Short Pitch 2024
Home Furnishings Ecommerce Platform Short Pitch 2024
 
Durg CALL GIRL ❤ 82729*64427❤ CALL GIRLS IN durg ESCORTS
Durg CALL GIRL ❤ 82729*64427❤ CALL GIRLS IN durg ESCORTSDurg CALL GIRL ❤ 82729*64427❤ CALL GIRLS IN durg ESCORTS
Durg CALL GIRL ❤ 82729*64427❤ CALL GIRLS IN durg ESCORTS
 
KOLKATA 💋 Call Girl 9827461493 Call Girls in Escort service book now
KOLKATA 💋 Call Girl 9827461493 Call Girls in  Escort service book nowKOLKATA 💋 Call Girl 9827461493 Call Girls in  Escort service book now
KOLKATA 💋 Call Girl 9827461493 Call Girls in Escort service book now
 

Ultimate_Analytical_by_Tanvir_Touhid.pdf

  • 2. Always write down whatever you’re studying 1. Draw diagrams for road, network etc. type puzzles 2. If it wants sequences of correct answers, take 1 constraint and then eliminate one option 3. Use the rough page given behind the question paper in the exam hall to do puzzles. 4. Tanvir Touhid
  • 3. Zebra Puzzle #1 Eight persons F, G, H, I, J, K, L and M attend 8 different seminars in four different months: January, April, November and December of the same year. Each seminar was conducted on either 15th or 25th of the month. G attends the seminar on the 15th of a month which has only 30 days. M and J both attend their seminars in the month before G, but not in the month of April. M attends the seminar after J. No one attends the seminar after F. Only one person attends the seminar between M and K. Both H and L attend the seminar on the 15th of different months. L does not attend the seminar in December. L and I attend the seminar in the same month. Tanvir Touhid
  • 4. 11. Who among the following attends the seminar on 15th Nov? (A). K (B). L (C). K (D). I (E). H 12. Which of the following combinations of Month-Date-Person is correct? (A). January – 15 – G (B). January – 25 – K (C). Nov – 25 – I (D). April – 15 – F (E). April – 25 – F 13. Which of the following statements is true with respect to the given arrangement? (A) G attends the seminar in November (B) H attends the seminar immediately after I. (C) M attends the seminar immediately before K. (D) K attends the seminar in November (E) None of the given statements is true 14. Who among the following attends a seminar in December? (A). F (B). M (C). G (D). K (E). J 15. How many people attend seminars between G and H? (A). One (B). Two (C). Three (D). Four (E). More than Four Tanvir Touhid
  • 5. Zebra Puzzle #2 (i) Five friends P, Q, R. S and T travelled to five different cities of Chennai, Calcutta, Delhi, Bangalore and Hyderabad by five different modes of transport of Bus. Train, Aeroplane, Car and Boat from Mumbai. (ii) The person who traveled to Delhi did not travel by boat. (iii) R went to Bangalore by car and Q went to Calcutta by aeroplane. (iv) S travelled by boat whereas T travelled by train. (v) Mumbai is not connected by bus to Delhi and Chennai. Tanvir Touhid
  • 6. Zebra Puzzle #2 1. Which of the following combinations of person and mode is not correct? (A) P — Bus (B) Q — Aeroplane (C) R — Car (D) S — Boat (E) T — Aeroplane 2. Which of the following combinations is true for S? (A) Delhi — Bus (B) Chennai — Bus (C) Chennai — Boat (D) Data inadequate (E) None of these 3. Which of the following combinations of place and mode is not correct? (A) Delhi — Bus (B) Calcutta — Aeroplane (C) Bangalore — Car (D) Chennai — Boat (E) Hyderabad — Bus Tanvir Touhid
  • 7. Zebra Puzzle #2 4. The person travelling to Delhi went by which of the following modes? (A) Bus (B) Train (C) Aeroplane (D) Car (E) Boat 5. Who among the following travelled to Delhi? (A) R (B) S (C) T (D) Data inadequate (E) None of these Tanvir Touhid
  • 8. Two circular dials of exactly the same size are pg 256 A chemist has exactly four unlabeled bottles pg 216 A cryptanalyst must translate into letters all of the digits pg 963 Seven pieces of equipment-an air compressor pg 884 An archaeologist is excavating a system of paths used by traders pg 489 In a city, the public transportation system consists of one subway line and one bus line pg 817 Internal audits in the Goodcrop Corporation pg 818 During its manufacture any widget pg 578 Difficult Puzzles Six horses-with the names "Greatness," " pg 763 A map is being prepared that will represent pg 687 A team of art historians has the task pg 549 Seven offices in an office building are to be painted pg 348 A student is planning his class schedule pg 293 A foundation has appointed two review panels pg 1043 Researchers are testing numerous water pg 1042 A certain kind of traditional cloth is woven pg 1022 Tanvir Touhid
  • 10. Tanvir Touhid Inference; are un-stated partial conclusions that can be drawn from the given premise. They do not contain the central idea, but they lead to/support the central idea. • An inference can serve as a link in the line of reasoning but is not the same as conclusion. A conclusion invariably addresses the central idea of the stimulus whereas inference serves only to support the conclusion • There can be many inferences; such as immediate inference, followed by final inference, before it leads to conclusion.
  • 11. Tanvir Touhid Premise #1: Students who get seven hours of sleep at night tend to be more alert the next day than those who don’t get seven hours of sleep Premise #2: The ability to get good scores in any competitive exam depends on one’s level of alertness. • Inference: From the above two we can infer that, “if you are a student (may not be true for others) you are likely to be more alert next morning if you get seven hours of sleep” • Conclusion: From the above two we can conclude that, “student (may not be true for others) wishing to do well in competitive exams should try and get at least seven hours of sleep on day before the exam)
  • 12. Tanvir Touhid Approaching / Solving Critical Reasoning Questions 1 Read the stimulus, little swiftly 2 Read the question & re-read the stimulus, little steadily 3 Identify the conclusion** 4 Separate the evidence from the conclusion 5 Re-arrange the premise and conclusion to get clear line of reasoning 6 Pre-phrase the answer 7 Attack the question
  • 13. Tanvir Touhid Jimmy is a doctor (premise) All doctors go to medical school.(premise) So jimmy went to medical school (conclusion) Basic
  • 14. Tanvir Touhid A. Assumptions • An assumption is merely an unstated (implied) premise. • In logically correct arguments which contain an assumption, the premise + assumption = conclusion. • An assumption bridges the gap between argument’s stated premises and conclusion. • Remember, since the assumption is an UNSTATED premise, any answer choice that comes from the passage to support your assumption is necessarily incorrect. • For assumption questions, find the conclusion and determine which answer choice needs to be true for a conclusion to be valid.(It must be a statement that completely supports the conclusion) Types
  • 15. Tanvir Touhid CORRECT ANSWER CHOICES: • Will be supporter or defender • Supporters help to link unrelated information presented in the stimulus and fill logical gaps • Defenders eliminate possibilities of weakness and attack to the stimulus/conclusion. That means, assumption questions:
  • 16. Tanvir Touhid Assumption Problem #1 For several years, Nighttime News attracted fewer viewers than World News, which broadcasts its show at the same time as Nighttime News. Recently,the producers of Nighttime News added personal interest stories and increased coverage of sports and weather. The two programs now have a roughly equal number of viewers. Clearly, the recent programming changes persuaded viewers to switch from World News to Nighttime News. Which of the following is an assumption on which the author relies? (D) There are other possible causes for an increase in the number of viewers of Nighttime News, including a recent ad campaign that aired on many local affiliates. (A) Viewers are more interested in sports and weather than in personal interest stories. B) The programming content of Nighttime News is more closely aligned with the interests of the overall audience than is the content of World News. (C) Some World News viewers liked the new Nighttime News programming better than they liked the World News programming. (E) The quality of World News will remain constant even if Nighttime News improves.
  • 17. Tanvir Touhid Assumption Problem #1 For several years, Nighttime News attracted fewer viewers than World News, which broadcasts its show at the same time as Nighttime News. Recently,the producers of Nighttime News added personal interest stories and increased coverage of sports and weather. The two programs no.w have a roughly equal number of viewers. Clearly, the recent programming changes persuaded viewers to switch from World News to Nighttime News. Which of the following is an assumption on which the author relies? (D) There are other possible causes for an increase in the number of viewers of Nighttime News, including a recent ad campaign that aired on many local affiliates. (A) Viewers are more interested in sports and weather than in personal interest stories. B) The programming content of Nighttime News is more closely aligned with the interests of the overall audience than is the content of World News. (C) Some World News viewers liked the new Nighttime News programming better than they liked the World News programming. (E) The quality of World News will remain constant even if Nighttime News improves.
  • 18. Tanvir Touhid Assumption Problem #2 Although the ratio of physicians to total population is about the same in the United States and Canada, the United States has 33 percent more surgeons per capita. Clearly, this is the reason people in the United States undergo 40 percent more operations per capita than do Canadians. The explanation given above rests on an assumption that: (D) General practitioners in the United States do not as a rule examine a who is a candidate for surgery before sending the patient to a surgeon. (A) patients in the United States do not have a greater need for surgery than do patients in Canada. (B) the population of the United States is not larger than that of Canada. (C) United States patients sometimes travel to Canada for certain kinds of surgery. (E) There are no unnecessary surgical operations performed in Canada.
  • 19. Tanvir Touhid Assumption Problem #2 Although the ratio of physicians to total population is about the same in the United States and Canada, the United States has 33 percent more surgeons per capita. Clearly, this is the reason people in the United States undergo 40 percent more operations per capita than do Canadians. The explanation given above rests on an assumption that: (D) General practitioners in the United States do not as a rule examine a who is a candidate for surgery before sending the patient to a surgeon. (A) patients in the United States do not have a greater need for surgery than do patients in Canada. (B) the population of the United States is not larger than that of Canada. (C) United States patients sometimes travel to Canada for certain kinds of surgery. (E) There are no unnecessary surgical operations performed in Canada.
  • 20. Tanvir Touhid Assumption Problem #3 Ramirez: The film industry claims that pirated DVDs, which are usually cheaper than legitimate DVDs and become available well before a film's official DVD release date, adversely affect its bottom line. But the industry should note what the spread of piracy indicates: consumers want lower prices and faster DVD releases. Lowering prices of DVDs and releasing them sooner would mitigate piracy's negative effect on film industry profits. The argument above relies on which of the following assumptions? D. Some current sellers of pirated DVDs would likely discontinue their businesses if legitimate DVDs were released faster and priced lower. A. Releasing legitimate DVDs earlier would not cause any reduction in the revenue the film industry receives from the films' theatrical release. B. Some people who would otherwise purchase pirated DVDs would be willing to purchase legitimate DVDs if they were less expensive and released earlier than they are now. C. The film industry will in the future be able to produce DVDs more cheaply than is currently the case. E. Current purchasers of pirated DVDs are aware that those DVDs are not authorized by the film industry.
  • 21. Tanvir Touhid Assumption Problem #3 Ramirez: The film industry claims that pirated DVDs, which are usually cheaper than legitimate DVDs and become available well before a film's official DVD release date, adversely affect its bottom line. But the industry should note what the spread of piracy indicates: consumers want lower prices and faster DVD releases. Lowering prices of DVDs and releasing them sooner would mitigate piracy's negative effect on film industry profits. The argument above relies on which of the following assumptions? D. Some current sellers of pirated DVDs would likely discontinue their businesses if legitimate DVDs were released faster and priced lower. A. Releasing legitimate DVDs earlier would not cause any reduction in the revenue the film industry receives from the films' theatrical release. B. Some people who would otherwise purchase pirated DVDs would be willing to purchase legitimate DVDs if they were less expensive and released earlier than they are now. C. The film industry will in the future be able to produce DVDs more cheaply than is currently the case. E. Current purchasers of pirated DVDs are aware that those DVDs are not authorized by the film industry.
  • 22. Tanvir Touhid Assumption Problem #4 Because no employee wants to be associated with bad news in the eyes of a superior, information about serious problems at lower levels is progressively softened and distorted as it goes up each step in the management hierarchy. The chief executive is therefore, less well informed about problems at lower levels than are his or her subordinates at those levels. The conclusion drawn above is based on the assumption that The conclusion drawn above is based on the assumption that (A) problems should be solved at levels in the management hierarchy at which they occur (B) employees should be rewarded for accurately reporting problems to their superiors (C) problems-solving ability is more important at higher levels than it is at lower levels of the management hierarchy (D) chief executives obtain information about problems at lower levels from the no source other than their subordinates (E) some employees are more concerned about truth than about the way they are perceived by their superiors
  • 23. Tanvir Touhid Assumption Problem #4 Because no employee wants to be associated with bad news in the eyes of a superior, information about serious problems at lower levels is progressively softened and distorted as it goes up each step in the management hierarchy. The chief executive is therefore, less well informed about problems at lower levels than are his or her subordinates at those levels. The conclusion drawn above is based on the assumption that The conclusion drawn above is based on the assumption that (A) problems should be solved at levels in the management hierarchy at which they occur (B) employees should be rewarded for accurately reporting problems to their superiors (C) problems-solving ability is more important at higher levels than it is at lower levels of the management hierarchy (D) chief executives obtain information about problems at lower levels from the no source other than their subordinates (E) some employees are more concerned about truth than about the way they are perceived by their superiors
  • 24. Tanvir Touhid Assumption Problem #5 Colleges in Tycho City have failed to prepare their students for the business world. A recent study revealed that the majority of college graduates in Tycho City could not write a simple business letter. Which of the following, if true, would provide additional evidence in support of the claim above? (A) A majority of students attending colleges in Tycho City are business majors. B) The State College in neighboring Twyla Township has recently improved its business program by adding courses in business writing. C) Most Tycho City college graduates move outside the Tycho City after they graduate. D) Most Tycho City college students live in on-campus dormitories. E) The majority of college graduates living in Tycho City received their college degrees from institutions located in Tycho city.
  • 25. Tanvir Touhid Assumption Problem #5 Colleges in Tycho City have failed to prepare their students for the business world. A recent study revealed that the majority of college graduates in Tycho City could not write a simple business letter. Which of the following, if true, would provide additional evidence in support of the claim above? (A) A majority of students attending colleges in Tycho City are business majors. B) The State College in neighboring Twyla Township has recently improved its business program by adding courses in business writing. C) Most Tycho City college graduates move outside the Tycho City after they graduate. D) Most Tycho City college students live in on-campus dormitories. E) The majority of college graduates living in Tycho City received their college degrees from institutions located in Tycho city.
  • 26. Tanvir Touhid B. Strengthen the Argument Identify the conclusion—this is what you are trying to strengthen! Find the logical gap and fix it with additional information. This is the ONLY type of GMAT question where additional information (outside of the question) can/should be used. Correct answers to this question type will: • Connect evidence with conclusion better • Make conclusion stronger. • Strengthen the evidence with new information (perhaps an assumption is needed to make the argument work) Types
  • 27. Tanvir Touhid Strengthen Problem #1 In an effort to make college education more affordable, the government of Grendania plans to subsidize college education by paying up to fifteen percent of college tuition for any Grendanian who attends college full time. However, rather than make college more affordable for students, implementing this plan will serve only to increase the revenue of universities. Which of the following most clearly supports the conclusion above? A. The government plans to leave in place programs that fund research engaged in by university professors. B. Universities in Grendania base the levels of their tuition on the amounts that people are willing to pay. C. When college education is more affordable, more people choose to attend college full time. D. In Grendania, the cost of attending college has been increasing at a rate much faster than the general rate of price inflation. E. Even if the average college tuition in Grendania were fifteen percent lower, many people would still have difficulty affording to attend college.
  • 28. Tanvir Touhid Strengthen Problem #1 In an effort to make college education more affordable, the government of Grendania plans to subsidize college education by paying up to fifteen percent of college tuition for any Grendanian who attends college full time. However, rather than make college more affordable for students, implementing this plan will serve only to increase the revenue of universities. Which of the following most clearly supports the conclusion above? A. The government plans to leave in place programs that fund research engaged in by university professors. B. Universities in Grendania base the levels of their tuition on the amounts that people are willing to pay. C. When college education is more affordable, more people choose to attend college full time. D. In Grendania, the cost of attending college has been increasing at a rate much faster than the general rate of price inflation. E. Even if the average college tuition in Grendania were fifteen percent lower, many people would still have difficulty affording to attend college.
  • 29. Tanvir Touhid Strengthen Problem #2 Mr. Blatt: Expert consultants are sought after by management because they help executives make better decisions. That is why they are worth the substantial fees they charge. Ms. Fring: Nonsense. Expert consultants are hired in order to enable executives to avoid responsibility. The more the experts cost, the more they can be blamed when things go wrong. Which one of the following, if it occurred, would be the strongest evidence favoring Ms. Fring’s opinion? (A) A company that is trying to decide whether to move its manufacturing plant hires an expensive expert to conduct a cost/benefit analysis. (B) Two competing companies faced with very similar problems adopt different solutions, one with the help of a consultant, one without. (C) A successful firm of expert consultants seeks to increase its volume of business by reducing its fees, but its volume of business drops. (D) An expert consultant builds up a successful business by charging clients a substantial percentage of the amount an independent assessor judges that the consultant saved the company. (E) A company follows a consultant’s advice to open two new stores, but both stores are only marginally profitable at first.
  • 30. Tanvir Touhid Strengthen Problem #2 Mr. Blatt: Expert consultants are sought after by management because they help executives make better decisions. That is why they are worth the substantial fees they charge. Ms. Fring: Nonsense. Expert consultants are hired in order to enable executives to avoid responsibility. The more the experts cost, the more they can be blamed when things go wrong. Which one of the following, if it occurred, would be the strongest evidence favoring Ms. Fring’s opinion? (A) A company that is trying to decide whether to move its manufacturing plant hires an expensive expert to conduct a cost/benefit analysis. (B) Two competing companies faced with very similar problems adopt different solutions, one with the help of a consultant, one without. (C) A successful firm of expert consultants seeks to increase its volume of business by reducing its fees, but its volume of business drops. (D) An expert consultant builds up a successful business by charging clients a substantial percentage of the amount an independent assessor judges that the consultant saved the company. (E) A company follows a consultant’s advice to open two new stores, but both stores are only marginally profitable at first.
  • 31. Tanvir Touhid Strengthen Problem #3 One summer, floods covered low-lying garlic fields situated in a region with a large mosquito population. Since mosquitoes lay their eggs in standing water, flooded fields would normally attract mosquitoes, yet no mosquitoes were found in the fields. Diallyl sulfide, a major component of garlic, is known to repel several species of insects, including mosquitoes, so it is likely that diallyl sulfide from the garlic repelled the mosquitoes. Which of the following, if true, most strengthens the argument? (A) Diallyl sulfide is also found in onions but at concentrations lower than in garlic. (B) The mosquito population of the region as a whole was significantly smaller during the year in which the flooding took place than it had been in previous years. (C) By the end of the summer, most of the garlic plants in the flooded fields had been killed by waterborne fungi. (D) Many insect species not repelled by diallyl sulfide were found in the flooded garlic fields throughout the summer. (E) Mosquitoes are known to be susceptible to toxins in plants other than garlic, such as marigolds.
  • 32. Tanvir Touhid Strengthen Problem #3 One summer, floods covered low-lying garlic fields situated in a region with a large mosquito population. Since mosquitoes lay their eggs in standing water, flooded fields would normally attract mosquitoes, yet no mosquitoes were found in the fields. Diallyl sulfide, a major component of garlic, is known to repel several species of insects, including mosquitoes, so it is likely that diallyl sulfide from the garlic repelled the mosquitoes. Which of the following, if true, most strengthens the argument? (A) Diallyl sulfide is also found in onions but at concentrations lower than in garlic. (B) The mosquito population of the region as a whole was significantly smaller during the year in which the flooding took place than it had been in previous years. (C) By the end of the summer, most of the garlic plants in the flooded fields had been killed by waterborne fungi. (D) Many insect species not repelled by diallyl sulfide were found in the flooded garlic fields throughout the summer. (E) Mosquitoes are known to be susceptible to toxins in plants other than garlic, such as marigolds.
  • 33. Tanvir Touhid C. Weaken The Argument To solve these questions, you first need to identify the premise and the conclusion. In this question type, we assume an answer choice presented to be true – even if it introduces new information (obviously, the information has to be relevant to the stimulus) ANSWER CHOICE QUALIFICATION: • Should rebuke the conclusion of the stimulus • Answer choices are taken to be true, even if there is new information provided. • Will either break down causality or show an obvious error in reasoning in formation of the conclusion CORRECT ANSWER CHOICES: • Will point out an obvious reason for the illogical conclusion • Enumerate a wrong generalization • Point out improper comparisons between two scenarios that the author assumed Types
  • 34. Tanvir Touhid Weaken Problem #1 Teenagers are often priced out of the labor market by the government-mandated minimum-wage level because employers cannot afford to pay that much for extra help. Therefore, if Congress institutes a subminimum wage, a new lower legal wage for teenagers, the teenage unemployment rate, which has been rising since 1960, will no longer increase. Which of the following, if true, would most weaken the argument above? (A) Since 1960 the teenage unemployment rate has risen when the minimum wage has risen. (B) Since 1960 the teenage unemployment rate has risen even when the minimum wage remained constant. (C) Employers often hire extra help during holiday and warm weather seasons. (D) The teenage unemployment rate rose more quickly in the 1970's than it did in the 1960's. (E) The teenage unemployment rate has occasionally declined in the years since 1960.
  • 35. Tanvir Touhid Weaken Problem #1 Teenagers are often priced out of the labor market by the government-mandated minimum-wage level because employers cannot afford to pay that much for extra help. Therefore, if Congress institutes a subminimum wage, a new lower legal wage for teenagers, the teenage unemployment rate, which has been rising since 1960, will no longer increase. Which of the following, if true, would most weaken the argument above? (A) Since 1960 the teenage unemployment rate has risen when the minimum wage has risen. (B) Since 1960 the teenage unemployment rate has risen even when the minimum wage remained constant. (C) Employers often hire extra help during holiday and warm weather seasons. (D) The teenage unemployment rate rose more quickly in the 1970's than it did in the 1960's. (E) The teenage unemployment rate has occasionally declined in the years since 1960.
  • 36. Tanvir Touhid Weaken Problem #2 Opponents of laws that require automobile drivers and passengers to wear seat belts argue that in a free society people have the right to take risks as long as people do not harm others as a result of taking the risks. As a result, they conclude that it should be each person’s decision whether or not to wear a seat belt. Which of the following, if true, most seriously weakens the conclusion drawn above? (A) Many new cars are built with seat belts that automatically fasten when someone sits in the front seat. (B) Automobile insurance rates for all automobile owners are higher because of the need to pay for the increased injuries or deaths of people not wearing seat belts. (C) Passengers in airplanes are required to wear seat belts during takeoffs and landings. (D) The rate of automobile fatalities in states that do not have mandatory seat belt laws is greater than the rate of fatalities in states that do have such laws. (E) In automobile accidents, a greater number of passengers who do not wear seat belts are injured than are passengers who do wear seat belts.
  • 37. Tanvir Touhid Weaken Problem #2 Opponents of laws that require automobile drivers and passengers to wear seat belts argue that in a free society people have the right to take risks as long as people do not harm others as a result of taking the risks. As a result, they conclude that it should be each person’s decision whether or not to wear a seat belt. Which of the following, if true, most seriously weakens the conclusion drawn above? (A) Many new cars are built with seat belts that automatically fasten when someone sits in the front seat. (B) Automobile insurance rates for all automobile owners are higher because of the need to pay for the increased injuries or deaths of people not wearing seat belts. (C) Passengers in airplanes are required to wear seat belts during takeoffs and landings. (D) The rate of automobile fatalities in states that do not have mandatory seat belt laws is greater than the rate of fatalities in states that do have such laws. (E) In automobile accidents, a greater number of passengers who do not wear seat belts are injured than are passengers who do wear seat belts.
  • 38. Tanvir Touhid Weaken Problem #3 A study of marital relationships in which one partner's sleeping and waking cycles differ from those of the other partner reveals that such couples share fewer activities with each other and have more violent arguments than do couples in a relationship in which both partners follow the same sleeping and waking patterns. Thus, mismatched sleeping and waking cycles can seriously jeopardize a marriage. Which of the following, if true, most seriously weakens the argument above? A) Married couples in which both spouses follow the same sleeping and waking patterns also occasionally have arguments that can jeopardize the couple's marriage. (B) The sleeping and waking cycles of individuals tend to vary from season to season. (C) The individuals who have sleeping and waking cycles that differ significantly from those of their spouses tend to argue little with colleagues at work. (D) People in unhappy marriages have been found to express hostility by adopting a different sleeping and waking cycle from that of their spouses. (E) According to a recent study, most people's sleeping and waking cycles can be controlled and modified easily.
  • 39. Tanvir Touhid Weaken Problem #3 A study of marital relationships in which one partner's sleeping and waking cycles differ from those of the other partner reveals that such couples share fewer activities with each other and have more violent arguments than do couples in a relationship in which both partners follow the same sleeping and waking patterns. Thus, mismatched sleeping and waking cycles can seriously jeopardize a marriage. Which of the following, if true, most seriously weakens the argument above? A) Married couples in which both spouses follow the same sleeping and waking patterns also occasionally have arguments that can jeopardize the couple's marriage. (B) The sleeping and waking cycles of individuals tend to vary from season to season. (C) The individuals who have sleeping and waking cycles that differ significantly from those of their spouses tend to argue little with colleagues at work. (D) People in unhappy marriages have been found to express hostility by adopting a different sleeping and waking cycle from that of their spouses. (E) According to a recent study, most people's sleeping and waking cycles can be controlled and modified easily.
  • 40. Tanvir Touhid D. MUST BE TRUE/ INFERENCE/ MAIN POINT/CONCLUSION Consider the evidence, draw a conclusion. An inference is an extension of an argument, not a necessary part of it. A valid inference is a conclusion, but not necessarily the conclusion, of a set of statements. For inference questions, determine which answer choice must absolutely, positively be true based on what you’ve read. • Pick the obvious answer choice. • Avoid extreme answers (too strong or too weak) CORRECT ANSWER CHOICES: • Restatement of the conclusion • Combination of one or more premises • Should be the main point of the stimulus, not just a premise (for Main Point questions- Repeat premises are wrong ) Types
  • 41. Tanvir Touhid Must-be-true Problem Transportation expenses accounted for a large portion of the total dollar amount spent on trips for pleasure by residents of the United States in 1997, and about half of the total dollar amount spent on the transportation was airfare. However, the large majority of United States residents who took trips for pleasure in 1997 did not travel by airplane but used other means of transportation. If the statements above are true, which of the following must also be true about United States residents who took trips for pleasure in 1997? (A) Most of those who traveled by airplane did so because the airfare to their destination was lower than the cost of other available means of transportation. (B) Most of those who traveled by airplane did so because other means of transportation to their destinations were unavailable. (C) Per mile traveled, those who traveled by airplane tended to spend more on transportation to their destination than did those who used other means of transportation. (D) Overall, people who did not travel by airplane had lower average transportation expenses than people who did. (E) Those who traveled by airplane spent about as much, on average, on other means of transportation as they did on airfare.
  • 42. Tanvir Touhid Must-be-true Problem Transportation expenses accounted for a large portion of the total dollar amount spent on trips for pleasure by residents of the United States in 1997, and about half of the total dollar amount spent on the transportation was airfare. However, the large majority of United States residents who took trips for pleasure in 1997 did not travel by airplane but used other means of transportation. If the statements above are true, which of the following must also be true about United States residents who took trips for pleasure in 1997? (A) Most of those who traveled by airplane did so because the airfare to their destination was lower than the cost of other available means of transportation. (B) Most of those who traveled by airplane did so because other means of transportation to their destinations were unavailable. (C) Per mile traveled, those who traveled by airplane tended to spend more on transportation to their destination than did those who used other means of transportation. (D) Overall, people who did not travel by airplane had lower average transportation expenses than people who did. (E) Those who traveled by airplane spent about as much, on average, on other means of transportation as they did on airfare.
  • 43. Tanvir Touhid Inference Problem In many school districts, part-time teachers are being laid off to save money. However, a decrease in the number of teachers generally results in a decrease in student performance. Thus, the same school districts that are laying off part-time teachers to save money will eventually have to hire more tutors to counteract diminished student performance. The author is arguing that A. diminished student performance will keep school districts from laying off part-time teachers B. laying off part-time teachers to save money will result in other costs to the school districts C. many part-time teachers who are laid off will have to retrain for other types of jobs D. school districts that are laying off teachers will eventually rehire many of them E. school districts will not save money by laying off part-time teachers
  • 44. Tanvir Touhid Inference Problem In many school districts, part-time teachers are being laid off to save money. However, a decrease in the number of teachers generally results in a decrease in student performance. Thus, the same school districts that are laying off part-time teachers to save money will eventually have to hire more tutors to counteract diminished student performance. The author is arguing that A. diminished student performance will keep school districts from laying off part-time teachers B. laying off part-time teachers to save money will result in other costs to the school districts C. many part-time teachers who are laid off will have to retrain for other types of jobs D. school districts that are laying off teachers will eventually rehire many of them E. school districts will not save money by laying off part-time teachers
  • 45. Tanvir Touhid Logical Flaw Cotrell is, at best, able to write magazine articles of average quality. The most compelling piece of evidence for this are those few of the numerous articles submitted by Cotrell that are superior, since Cotrell, who is incapable of writing an article that is better than average, must obviously have plagiarized superior ones. The argument is most vulnerable to criticism on which one of the following grounds? (A) It simply ignores the existence of potential counterevidence. (B) It generalizes from atypical occurrences. (C) It presupposes what it seeks to establish (D) It relies on the judgement of experts in a matter to which their expertise is irrelevant (E) It infers limits on ability from a few isolated lapses in performance
  • 46. Tanvir Touhid Logical Flaw Cotrell is, at best, able to write magazine articles of average quality. The most compelling piece of evidence for this are those few of the numerous articles submitted by Cotrell that are superior, since Cotrell, who is incapable of writing an article that is better than average, must obviously have plagiarized superior ones. The argument is most vulnerable to criticism on which one of the following grounds? (A) It simply ignores the existence of potential counterevidence. (B) It generalizes from atypical occurrences. (C) It presupposes what it seeks to establish (D) It relies on the judgement of experts in a matter to which their expertise is irrelevant (E) It infers limits on ability from a few isolated lapses in performance
  • 47. Tanvir Touhid Complete The Passage Exporters in Country X are facing lower revenues due to a shortage of the large metal shipping containers in which they send their goods by sea to other countries. Fewer containers arrive in Country X due to reductions in imports. This has meant lost orders, costly delays, and a scramble for alternatives, such as air freight, all of which are costlier. Moreover, the revenues of exporters in Country X will probably continue to decline in the near future. This is because other countries are likely to find it increasingly unprofitable to export their goods to Country X, and because __________. A. production of shipping containers in Country X is growing rapidly as a response to the shortage B. shipping companies are willing to move containers from country to country only when the containers are full C. the cost of shipping alternatives such as air freight is likely to stabilize in the near future D. consumers in Country X are purchasing more products than ever before E. the worldwide demand for goods made in Country X has only recently begun to rise after a long decline
  • 48. Tanvir Touhid Complete The Passage Exporters in Country X are facing lower revenues due to a shortage of the large metal shipping containers in which they send their goods by sea to other countries. Fewer containers arrive in Country X due to reductions in imports. This has meant lost orders, costly delays, and a scramble for alternatives, such as air freight, all of which are costlier. Moreover, the revenues of exporters in Country X will probably continue to decline in the near future. This is because other countries are likely to find it increasingly unprofitable to export their goods to Country X, and because __________. A. production of shipping containers in Country X is growing rapidly as a response to the shortage B. shipping companies are willing to move containers from country to country only when the containers are full C. the cost of shipping alternatives such as air freight is likely to stabilize in the near future D. consumers in Country X are purchasing more products than ever before E. the worldwide demand for goods made in Country X has only recently begun to rise after a long decline
  • 49. Tanvir Touhid Bold face CR Public health expert: Increasing the urgency of a public health message may be counterproductive. In addition to irritating the majority who already behave responsibly, it may undermine all government pronouncements on health by convincing people that such messages are overly cautious. And there is no reason to believe that those who ignore measured voices will listen to shouting. The two sections in boldface play which of the following roles in the public health expert's argument? (A) The first is a conclusion for which support is provided. but is not the argument's main conclusion; the second is an unsupported premise supporting the arguments main conclusion. (B) The first is a premise supporting the only explicit conclusion; so is the second. (C) The first is the argument's main conclusion; the second supports that conclusion and is itself a conclusion for which support is provided. (D) The first is a premise supporting the argument's only conclusion; the second is that conclusion. (E) The first is the argument‘s only explicit conclusion; the second is a premise supporting that conclusion.
  • 50. Tanvir Touhid Bold face CR Public health expert: Increasing the urgency of a public health message may be counterproductive. In addition to irritating the majority who already behave responsibly, it may undermine all government pronouncements on health by convincing people that such messages are overly cautious. And there is no reason to believe that those who ignore measured voices will listen to shouting. The two sections in boldface play which of the following roles in the public health expert's argument? (A) The first is a conclusion for which support is provided. but is not the argument's main conclusion; the second is an unsupported premise supporting the arguments main conclusion. (B) The first is a premise supporting the only explicit conclusion; so is the second. (C) The first is the argument's main conclusion; the second supports that conclusion and is itself a conclusion for which support is provided. (D) The first is a premise supporting the argument's only conclusion; the second is that conclusion. (E) The first is the argument‘s only explicit conclusion; the second is a premise supporting that conclusion.
  • 51. Tanvir Touhid E. Resolve the Paradox To solve this type of question, look for a logically contradictory discrepancy. • Often the correct answer will take a similar format (in terms of answer length or argument structure). Types
  • 52. Tanvir Touhid Resolve the Paradox #1 A certain type of insect trap uses a scented lure to attract rose beetles into a plastic bag from which it is difficult for them to escape. If several of these traps are installed in a backyard garden, the number of rose beetles in the garden will be greatly reduced. If only one trap is installed, however, the number of rose beetles in the garden will actually increase. Which one of the following, if true, most helps to resolve the apparent discrepancy? (A) The scent of a single trap’s lure usually cannot be detected throughout a backyard garden by rose beetles. (B) Several traps are better able to catch a large number of rose beetles than is one trap alone, since any rose beetles that evade one trap are likely to encounter another trap if there are several traps in the garden. (C) When there are several traps in a garden, they each capture fewer rose beetles than any single trap would if it were the only trap in the garden. (D) The presence of any traps in a backyard garden will attract more rose beetles than one trap can catch, but several traps will not attract significantly more rose beetles to a garden than one trap will. (E) When there is only one trap in the garden, the plastic bag quickly becomes filled to capacity, allowing some rose beetles to escape.
  • 53. Tanvir Touhid Resolve the Paradox #1 A certain type of insect trap uses a scented lure to attract rose beetles into a plastic bag from which it is difficult for them to escape. If several of these traps are installed in a backyard garden, the number of rose beetles in the garden will be greatly reduced. If only one trap is installed, however, the number of rose beetles in the garden will actually increase. Which one of the following, if true, most helps to resolve the apparent discrepancy? (A) The scent of a single trap’s lure usually cannot be detected throughout a backyard garden by rose beetles. (B) Several traps are better able to catch a large number of rose beetles than is one trap alone, since any rose beetles that evade one trap are likely to encounter another trap if there are several traps in the garden. (C) When there are several traps in a garden, they each capture fewer rose beetles than any single trap would if it were the only trap in the garden. (D) The presence of any traps in a backyard garden will attract more rose beetles than one trap can catch, but several traps will not attract significantly more rose beetles to a garden than one trap will. (E) When there is only one trap in the garden, the plastic bag quickly becomes filled to capacity, allowing some rose beetles to escape.
  • 54. Tanvir Touhid Resolve the Paradox #2 Technological improvements and reduced equipment costs have made converting solar energy directly into electricity far more cost-efficient in the last decade. However, the threshold of economic viability for solar power (that is, the price per barrel to which oil would have to rise in order for new solar power plants to be more economical than new oil-fired power plants) is unchanged at thirty-five dollars. Which of the following, if true, does most to help explain why the increased cost- efficiency of solar power has not decreased its threshold of economic viability? (A) The cost of oil has fallen dramatically. (B) The reduction in the cost of solar-power equipment has occurred despite increased raw material costs for that equipment. (C) Technological changes have increased the efficiency of oil-fired power plants. (D) Most electricity is generated by coal-fired or nuclear, rather than oil-fired, power plants. (E) When the price of oil increases, reserves of oil not previously worth exploiting become economically viable.
  • 55. Tanvir Touhid Resolve the Paradox #2 Technological improvements and reduced equipment costs have made converting solar energy directly into electricity far more cost-efficient in the last decade. However, the threshold of economic viability for solar power (that is, the price per barrel to which oil would have to rise in order for new solar power plants to be more economical than new oil-fired power plants) is unchanged at thirty-five dollars. Which of the following, if true, does most to help explain why the increased cost- efficiency of solar power has not decreased its threshold of economic viability? (A) The cost of oil has fallen dramatically. (B) The reduction in the cost of solar-power equipment has occurred despite increased raw material costs for that equipment. (C) Technological changes have increased the efficiency of oil-fired power plants. (D) Most electricity is generated by coal-fired or nuclear, rather than oil-fired, power plants. (E) When the price of oil increases, reserves of oil not previously worth exploiting become economically viable.
  • 56. Tanvir Touhid F. Evaluate the Argument -The question stem will likely use some form of the word “evaluate,” “determine,” or “useful / important to know.” -Find the conclusion and identify the main supporting premise. If you can articulate any gaps between the main premise and the conclusion, do so. Think of the conclusion as the author’s hypothesis; your job is to find the answer choice that can test that hypothesis. The answers will provide a “whether or not” type of statement, indicating information that might or might not be so. The correct answer will be able to both slightly strengthen and slightly weaken the hypothesis, depending upon whether the information in that choice is true or is not true. Types
  • 57. Tanvir Touhid Evaluate the Argument #1 Psychologist: In a survey, several hundred volunteers rated their own levels of self- control and their overall life satisfaction. The volunteers who rated themselves as having better self-control also reported greater satisfaction with their lives. This suggests that self-control is one factor that helps people avoid situations likely to produce dissatisfaction. In order to assess the strength of the psychologist's argument, it would be most helpful to know whether A. people typically rate themselves as having significantly better self-control than expert psychological assessments would rate them as having B. people's perceptions of how satisfied they are with their lives could be affected by factors of which they are unaware C. there is a high level of self-control that tends to reduce overall life satisfaction D. people's ratings of their overall satisfaction with their lives tend to temporarily decrease in situations likely to produce dissatisfaction E. feelings of dissatisfaction significantly interfere with people's ability to exercise self-control
  • 58. Tanvir Touhid Evaluate the Argument #1 Psychologist: In a survey, several hundred volunteers rated their own levels of self- control and their overall life satisfaction. The volunteers who rated themselves as having better self-control also reported greater satisfaction with their lives. This suggests that self-control is one factor that helps people avoid situations likely to produce dissatisfaction. In order to assess the strength of the psychologist's argument, it would be most helpful to know whether A. people typically rate themselves as having significantly better self-control than expert psychological assessments would rate them as having B. people's perceptions of how satisfied they are with their lives could be affected by factors of which they are unaware C. there is a high level of self-control that tends to reduce overall life satisfaction D. people's ratings of their overall satisfaction with their lives tend to temporarily decrease in situations likely to produce dissatisfaction E. feelings of dissatisfaction significantly interfere with people's ability to exercise self-control
  • 59. Tanvir Touhid Evaluate the Argument #2 Most of the world's supply of uranium currently comes from the mines. It is possible to extract uranium from seawater, but the cost of doing so is greater than the price that Uranium fetches on the world market. Therefore, until the cost of extracting uranium from seawater can somehow be reduced, this method of obtaining uranium is unlikely to be commercially viable. Which of the following would it be most useful to determine in evaluating the argument? (A) Whether the uranium in deposits on land is rapidly being depleted (B) Whether most uranium is used near where it is mined (C) Whether there are any technological advances that show promise of reducing the costs of extracting uranium from seawater (D) Whether the total amount of Uranium in seawater is significantly greater than the total amount of uranium on land (E) Whether uranium can be extracted from freshwater at a cost similar to the cost of extracting it from seawater
  • 60. Tanvir Touhid Evaluate the Argument #2 Most of the world's supply of uranium currently comes from the mines. It is possible to extract uranium from seawater, but the cost of doing so is greater than the price that Uranium fetches on the world market. Therefore, until the cost of extracting uranium from seawater can somehow be reduced, this method of obtaining uranium is unlikely to be commercially viable. Which of the following would it be most useful to determine in evaluating the argument? (A) Whether the uranium in deposits on land is rapidly being depleted (B) Whether most uranium is used near where it is mined (C) Whether there are any technological advances that show promise of reducing the costs of extracting uranium from seawater (D) Whether the total amount of Uranium in seawater is significantly greater than the total amount of uranium on land (E) Whether uranium can be extracted from freshwater at a cost similar to the cost of extracting it from seawater
  • 62. XY=? (1) X-Y=2 (2) X+Y=6 A – (1) ALONE is sufficient, but (2) alone is not sufficient. B – (2) ALONE is sufficient, but (1) alone is not sufficient. C – TOGETHER are sufficient, but NEITHER ALONE is sufficient. D – EACH ALONE is sufficient. E – NEITHER ALONE NOR TOGETHER is the statements sufficient. Tanvir Touhid
  • 63. #1- Is |x - y| > |x + y|? (1) x^2 - y^2 = 9 (2) x - y = 2 Tanvir Touhid
  • 64. #1- Is |x - y| > |x + y|? (1) x^2 - y^2 = 9 (2) x - y = 2 A – (1) ALONE is sufficient, but (2) alone is not sufficient. B – (2) ALONE is sufficient, but (1) alone is not sufficient. C – TOGETHER are sufficient, but NEITHER ALONE is sufficient. D – EACH ALONE is sufficient. E – NEITHER ALONE NOR TOGETHER is the statements sufficient. Tanvir Touhid
  • 65. #2- If x is a positive number less than 10, is z greater than the average (arithmetic mean) of x and 10? (1) On the number line, z is closer to 10 than it is to x. (2) z = 5x Tanvir Touhid
  • 66. #2- If x is a positive number less than 10, is z greater than the average (arithmetic mean) of x and 10? (1) On the number line, z is closer to 10 than it is to x. (2) z = 5x A – (1) ALONE is sufficient, but (2) alone is not sufficient. B – (2) ALONE is sufficient, but (1) alone is not sufficient. C – TOGETHER are sufficient, but NEITHER ALONE is sufficient. D – EACH ALONE is sufficient. E – NEITHER ALONE NOR TOGETHER is the statements sufficient. Tanvir Touhid
  • 67. #3- Amir sold a pen and a pencil in his shop. If both pen and pencil are sold at a certain profit, then the profit earned on pen is what percent of the profit earned on pencil? (1) Cost price of pen and cost price of pencil is in the ratio of 5:1. (2) The Pen is sold for $3.5 while the pencil is sold for $0.7. Tanvir Touhid
  • 68. #3- Amir sold a pen and a pencil in his shop. If both pen and pencil are sold at a certain profit, then the profit earned on pen is what percent of the profit earned on pencil? (1) Cost price of pen and cost price of pencil is in the ratio of 5:1. (2) The Pen is sold for $3.5 while the pencil is sold for $0.7. A – (1) ALONE is sufficient, but (2) alone is not sufficient. B – (2) ALONE is sufficient, but (1) alone is not sufficient. C – TOGETHER are sufficient, but NEITHER ALONE is sufficient. D – EACH ALONE is sufficient. E – NEITHER ALONE NOR TOGETHER is the statements sufficient. Tanvir Touhid
  • 69. #4- A team is composed of exactly 4 executives and 3 associates. How many such teams can be formed? (1) If two fewer executives were available for selection there would be an equal number of executives and associates available for selection. (2) If two more associates were available for selection exactly 10 different teams of 3 associates could be selected. Tanvir Touhid
  • 70. #4- A team is composed of exactly 4 executives and 3 associates. How many such teams can be formed? (1) If two fewer executives were available for selection there would be an equal number of executives and associates available for selection. (2) If two more associates were available for selection exactly 10 different teams of 3 associates could be selected. A – (1) ALONE is sufficient, but (2) alone is not sufficient. B – (2) ALONE is sufficient, but (1) alone is not sufficient. C – TOGETHER are sufficient, but NEITHER ALONE is sufficient. D – EACH ALONE is sufficient. E – NEITHER ALONE NOR TOGETHER is the statements sufficient. Tanvir Touhid
  • 71. #5- During a 40-mile trip, Marla traveled at an average speed of x miles per hour for the first y miles of the trip and and at an average speed of 1.25x miles per hour for the last 40 - y miles of the trip. The time that Marla took to travel the 40 miles was what percent of the time it would have taken her if she had traveled at an average speed of x miles per hour for the entire trip? (1) x = 48. (2) y = 20. Tanvir Touhid
  • 72. #5- During a 40-mile trip, Marla traveled at an average speed of x miles per hour for the first y miles of the trip and and at an average speed of 1.25x miles per hour for the last 40 - y miles of the trip. The time that Marla took to travel the 40 miles was what percent of the time it would have taken her if she had traveled at an average speed of x miles per hour for the entire trip? (1) x = 48. (2) y = 20. A – (1) ALONE is sufficient, but (2) alone is not sufficient. B – (2) ALONE is sufficient, but (1) alone is not sufficient. C – TOGETHER are sufficient, but NEITHER ALONE is sufficient. D – EACH ALONE is sufficient. E – NEITHER ALONE NOR TOGETHER is the statements sufficient. Tanvir Touhid
  • 73. #6- If 75 percent of the guests at a certain banquet ordered dessert, what percent of the guests ordered coffee? (1) 60 percent of the guests who ordered dessert also ordered coffee. (2) 90 percent of the guests who ordered coffee also ordered dessert. Tanvir Touhid
  • 74. #6- If 75 percent of the guests at a certain banquet ordered dessert, what percent of the guests ordered coffee? (1) 60 percent of the guests who ordered dessert also ordered coffee. (2) 90 percent of the guests who ordered coffee also ordered dessert. A – (1) ALONE is sufficient, but (2) alone is not sufficient. B – (2) ALONE is sufficient, but (1) alone is not sufficient. C – TOGETHER are sufficient, but NEITHER ALONE is sufficient. D – EACH ALONE is sufficient. E – NEITHER ALONE NOR TOGETHER is the statements sufficient. Tanvir Touhid
  • 75. #7- The average (arithmetic mean) of a list of positive numbers is what percent of the sum of the numbers? (1) The sum of the numbers in the list is 150. (2) There are 25 numbers in the list Tanvir Touhid
  • 76. #7- The average (arithmetic mean) of a list of positive numbers is what percent of the sum of the numbers? (1) The sum of the numbers in the list is 150. (2) There are 25 numbers in the list A – (1) ALONE is sufficient, but (2) alone is not sufficient. B – (2) ALONE is sufficient, but (1) alone is not sufficient. C – TOGETHER are sufficient, but NEITHER ALONE is sufficient. D – EACH ALONE is sufficient. E – NEITHER ALONE NOR TOGETHER is the statements sufficient. Tanvir Touhid
  • 77. #8- Is x > 10^10 ? (1) x > 2^34 (2) x = 2^35 Tanvir Touhid
  • 78. #8- Is x > 10^10 ? (1) x > 2^34 (2) x = 2^35 A – (1) ALONE is sufficient, but (2) alone is not sufficient. B – (2) ALONE is sufficient, but (1) alone is not sufficient. C – TOGETHER are sufficient, but NEITHER ALONE is sufficient. D – EACH ALONE is sufficient. E – NEITHER ALONE NOR TOGETHER is the statements sufficient. Tanvir Touhid
  • 79. #9- If x is an integer, is 2^x a factor of 12! ? (1) x is the sum of two distinct single-digit prime numbers. (2) 0 < x < 11 Tanvir Touhid
  • 80. #9- If x is an integer, is 2^x a factor of 12! ? (1) x is the sum of two distinct single-digit prime numbers. (2) 0 < x < 11 A – (1) ALONE is sufficient, but (2) alone is not sufficient. B – (2) ALONE is sufficient, but (1) alone is not sufficient. C – TOGETHER are sufficient, but NEITHER ALONE is sufficient. D – EACH ALONE is sufficient. E – NEITHER ALONE NOR TOGETHER is the statements sufficient. Tanvir Touhid
  • 81. #10-If x and y are integers and xy ≠ 0, is x - y > 0 ? (1) x/y < 1/2 (2) Tanvir Touhid
  • 82. #10-If x and y are integers and xy ≠ 0, is x - y > 0 ? (1) x/y < 1/2 (2) A – (1) ALONE is sufficient, but (2) alone is not sufficient. B – (2) ALONE is sufficient, but (1) alone is not sufficient. C – TOGETHER are sufficient, but NEITHER ALONE is sufficient. D – EACH ALONE is sufficient. E – NEITHER ALONE NOR TOGETHER is the statements sufficient. Tanvir Touhid
  • 83. #11-On the number line, the distance between x and y is greater than the distance between x and z. Does z lie between x and y on the number line? (1) xyz < 0 (2) xy < 0 Tanvir Touhid
  • 84. #11-On the number line, the distance between x and y is greater than the distance between x and z. Does z lie between x and y on the number line? (1) xyz < 0 (2) xy < 0 A – (1) ALONE is sufficient, but (2) alone is not sufficient. B – (2) ALONE is sufficient, but (1) alone is not sufficient. C – TOGETHER are sufficient, but NEITHER ALONE is sufficient. D – EACH ALONE is sufficient. E – NEITHER ALONE NOR TOGETHER is the statements sufficient. Tanvir Touhid
  • 85. #12-A paint mixture was formed by mixing exactly 3 colors of paint. By volume, the mixture was x% blue paint, y% green paint, and z% red paint. If exactly 1 gallon of blue paint and 3 gallons of red paint were used, how many gallons of green paint were used? (1) x = y (2) z = 60 Tanvir Touhid
  • 86. #12-A paint mixture was formed by mixing exactly 3 colors of paint. By volume, the mixture was x% blue paint, y% green paint, and z% red paint. If exactly 1 gallon of blue paint and 3 gallons of red paint were used, how many gallons of green paint were used? (1) x = y (2) z = 60 A – (1) ALONE is sufficient, but (2) alone is not sufficient. B – (2) ALONE is sufficient, but (1) alone is not sufficient. C – TOGETHER are sufficient, but NEITHER ALONE is sufficient. D – EACH ALONE is sufficient. E – NEITHER ALONE NOR TOGETHER is the statements sufficient. Tanvir Touhid
  • 87. #13-The positive integers x, y, and z are such that x is a factor of y and y is a factor of z. Is z even? (1) xz is even (2) y is even. Tanvir Touhid
  • 88. #13-The positive integers x, y, and z are such that x is a factor of y and y is a factor of z. Is z even? (1) xz is even (2) y is even. A – (1) ALONE is sufficient, but (2) alone is not sufficient. B – (2) ALONE is sufficient, but (1) alone is not sufficient. C – TOGETHER are sufficient, but NEITHER ALONE is sufficient. D – EACH ALONE is sufficient. E – NEITHER ALONE NOR TOGETHER is the statements sufficient. Tanvir Touhid
  • 89. #14-Water in a flooded basement is to be pumped into an empty right circular cylindrical container. If the height of the container is 250 cm, is the volume of the container sufficient for it to hold all the water? (1) The area of the flooded portion of the basement is 20 square meters. (2) The circumference of the container is 100 centimeters Tanvir Touhid
  • 90. #14-Water in a flooded basement is to be pumped into an empty right circular cylindrical container. If the height of the container is 250 cm, is the volume of the container sufficient for it to hold all the water? (1) The area of the flooded portion of the basement is 20 square meters. (2) The circumference of the container is 100 centimeters A – (1) ALONE is sufficient, but (2) alone is not sufficient. B – (2) ALONE is sufficient, but (1) alone is not sufficient. C – TOGETHER are sufficient, but NEITHER ALONE is sufficient. D – EACH ALONE is sufficient. E – NEITHER ALONE NOR TOGETHER is the statements sufficient. Tanvir Touhid
  • 91. #15-A real estate agent sold 6 houses in Junction City and 3 houses in Uniontown. For each house he sold, he earned a commission of 7% of the selling price of the house. Was the total of his commissions for the houses he sold in Junction City greater than the total of his commissions for the houses he sold in Uniontown? (1) The average (arithmetic mean) selling price of the 3 houses in Uniontown was more than twice the average selling price of the 6 houses in Junction City. (2) The selling price of each of the 3 houses in Uniontown was more than twice the selling price of the most expensive of the 6 houses in Junction City. Tanvir Touhid
  • 92. #15-A real estate agent sold 6 houses in Junction City and 3 houses in Uniontown. For each house he sold, he earned a commission of 7% of the selling price of the house. Was the total of his commissions for the houses he sold in Junction City greater than the total of his commissions for the houses he sold in Uniontown? (1) The average (arithmetic mean) selling price of the 3 houses in Uniontown was more than twice the average selling price of the 6 houses in Junction City. (2) The selling price of each of the 3 houses in Uniontown was more than twice the selling price of the most expensive of the 6 houses in Junction City. A – (1) ALONE is sufficient, but (2) alone is not sufficient. B – (2) ALONE is sufficient, but (1) alone is not sufficient. C – TOGETHER are sufficient, but NEITHER ALONE is sufficient. D – EACH ALONE is sufficient. E – NEITHER ALONE NOR TOGETHER is the statements sufficient. Tanvir Touhid
  • 93. #16-In a survey of retailers, what percent had purchased computers for business purposes? (1) 85 percent of the retailers surveyed who owned their own store had purchased computers for business purposes. (2) 40 percent of the retailers surveyed owned their own store. Tanvir Touhid
  • 94. #16-In a survey of retailers, what percent had purchased computers for business purposes? (1) 85 percent of the retailers surveyed who owned their own store had purchased computers for business purposes. (2) 40 percent of the retailers surveyed owned their own store. A – (1) ALONE is sufficient, but (2) alone is not sufficient. B – (2) ALONE is sufficient, but (1) alone is not sufficient. C – TOGETHER are sufficient, but NEITHER ALONE is sufficient. D – EACH ALONE is sufficient. E – NEITHER ALONE NOR TOGETHER is the statements sufficient. Tanvir Touhid
  • 95. #17-Seven different numbers are selected from the integers 1 to 100, and each number is divided by 7. What is the sum of the remainders? (1) The range of the seven remainders is 6. (2) The seven numbers selected are consecutive integers. Tanvir Touhid
  • 96. #17-Seven different numbers are selected from the integers 1 to 100, and each number is divided by 7. What is the sum of the remainders? (1) The range of the seven remainders is 6. (2) The seven numbers selected are consecutive integers. A – (1) ALONE is sufficient, but (2) alone is not sufficient. B – (2) ALONE is sufficient, but (1) alone is not sufficient. C – TOGETHER are sufficient, but NEITHER ALONE is sufficient. D – EACH ALONE is sufficient. E – NEITHER ALONE NOR TOGETHER is the statements sufficient. Tanvir Touhid
  • 97. #18-Is x > 1? (1) (x+1)(|x|−1)>0 (2) |x|<5 Tanvir Touhid
  • 98. #18-Is x > 1? (1) (x+1)(|x|−1)>0 (2) |x|<5 A – (1) ALONE is sufficient, but (2) alone is not sufficient. B – (2) ALONE is sufficient, but (1) alone is not sufficient. C – TOGETHER are sufficient, but NEITHER ALONE is sufficient. D – EACH ALONE is sufficient. E – NEITHER ALONE NOR TOGETHER is the statements sufficient. Tanvir Touhid